Cost Final Exam

¡Supera tus tareas y exámenes ahora con Quizwiz!

Crandle Manufacturers Inc. is approached by a potential customer to fulfill a one-time-only special order for a product similar to one offered to domestic customers. The company has excess capacity. The following per unit data apply for sales to regular customers Variable costs: Direct materials $130 Direct labor 110 Manufacturing support 125 Marketing costs 65 Fixed costs: Manufacturing support 175 Marketing costs 85 Total costs 690 Markup (50%) 345 Targeted selling price $1,035 What is the full cost of the product per unit?

690

The Speedjet Aircraft Corporation has a central materials laboratory. The laboratory has only two users, the Large Plane Department and the Small Plane Department. The following data apply to the coming budget year: Budgeted costs of operating the materials laboratory for 150,000 to 250,000 technician hours per year: Fixed costs per year $8,400,000 Variable costs $68​ per technician hourBudgeted long-run usage in hours per year: Large Plane Department 90,000​ technician hours Small Plane Department 160,000​ technician hours Budgeted amounts are used to calculate the allocation rates. Actual usage for the year by the Large Plane Department was 70,000 technician hours and by the Small Plane Department was 90,000 technician hours. If a single-rate cost-allocation method is used, what is the allocation rate per hour used? (Round the final answer to the nearest dollar.)

8400000/250000=33.6+68=101.6

Put the following steps in order for using the high-low method of estimating a cost function: A = Identify the cost function B = Calculate the constant C = Calculate the slope coefficient D = Identify the highest and lowest observed values A D C B C D A B D C A B

A D C B

Which of the following statements is true of direct costs? A direct cost of one cost object can be an indirect cost of another cost object. A direct cost of one cost object is a true sense of the budgeted costs. All variable costs are direct costs. All fixed costs are direct costs.

A direct cost of one cost object can be an indirect cost of another cost object.

Mendel Company makes the following journal entry: Variable Manufacturing Overhead Allocated 229,000 Variable Manufacturing Overhead Efficiency Variance 7,000 Variable Manufacturing Overhead Control 179,000 Variable Manufacturing Overhead Spending Variance 57,000 Which of the following statements is true of the given journal entry? A favorable flexible-budget variance of $50,000 is recorded. An unfavorable spending variance of $57,000 is recorded. A favorable efficiency variance of $7,000 is recorded. A variable manufacturing overhead cost of $179,000 is written-off.

A favorable flexible-budget variance of $50,000 is recorded.

Which of the following reasons explain why ABC concepts may be of great value to service companies? It promotes more accurate cost of goods sold reporting It allows for more accurate cost accounting for inventory A significant portion of the cost structure of a service firm is composed of indirect costs It helps make financial reporting more accurate

A significant portion of the cost structure of a service firm is composed of indirect costs

Which of the following statements about actual costing is true? Actual costing uses actual indirect-cost rates calculated annually. Manufacturing costs of a job are available earlier under actual costing. Actual costing uses budgeted indirect-cost rates calculated annually. Corrective actions can be implemented sooner under actual costing.

Actual costing uses actual indirect-cost rates calculated annually.

Place the following steps in order for estimating a cost function using quantitative analysis. A = Plot the data B = Collect data on the dependent variable and the cost driver. C = Choose the dependent variable D = Identify the independent variable, or cost driver E = Estimate the cost function C D B A E D C E A B E D C B A A D C E B

C D B A E

A common classification of costs is by business function. Which of the following classifications is not by business function? distribution cost object research and development production

Cost object

Which of the following would be subtracted from sales while calculating contribution margin in a variable costing format of an operating income statement? Direct labor in factory Sales commission on incremental sales Rent on the headquarters building Rent on factory building

Direct labor in factory

Which of the following is true of flexible budget? It calculates total variable cost by multiplying actual units by budgeted variable cost per unit. It calculates contribution margin by multiplying budgeted units by actual contribution margin per unit. It calculates revenues by multiplying budgeted units by actual selling price per unit. It calculates total fixed cost by multiplying actual units by budgeted fixed cost per unit.

It calculates total variable cost by multiplying actual units by budgeted variable cost per unit.

Which of the following is true of absorption costing? It enables a manager to decrease margins and operating income by producing more beginning inventory. It enables a manager to increase margins and operating income by producing more and building ending inventory. It enables a manager to decrease margins and operating income by producing more and building ending inventory. It enables a manager to increase margins and operating income by producing more beginning inventory.

It enables a manager to increase margins and operating income by producing more and building ending inventory.

Which of the following statements regarding manufacturing overhead allocation is true? Allocated amounts are debited to Manufacturing Overhead Control. The costs can be grouped only as a single indirect-cost pool. Total costs are unknown at the end of the accounting period. It includes all manufacturing costs that cannot be directly traced to a product or service.

It includes all manufacturing costs that cannot be directly traced to a product or service.

Which of the following is true about the assumptions underlying basic CVP analysis? Only selling price and variable cost per unit are known and constant. Selling price, variable cost per unit, fixed cost per unit, and total fixed costs are known and constant. Only selling price, variable cost per unit, and total fixed costs are known and constant. Selling price varies with demand and supply of the product.

Only selling price, variable cost per unit, and total fixed costs are known and constant.

Kanga Company is considering two different production plans. Option one: Fixed costs of $10,000 and a breakeven point of 500 units. Option two: Fixed costs of $20,000 and a breakeven point of 700 units. Which option should Kanga choose if it is expecting to produce 600 units? Option two. It isn't possible to determine from the information given. Option one. Both options are equally good.

Option one.

Which of the following best describes how fixed cost are treated in a variable cost method? They are excluded from inventory cost and are treated as period costs They are classified as nonmanufacturing costs They are allocated to the product cost using a denominator-level capacity choice They are part of the product cost

They are excluded from inventory cost and are treated as period costs

Which of the following is an assumption of CVP analysis? Total costs can be divided into inventoriable and period costs with respect to the level of output. The unit-selling price is variable as it is subject to demand and supply. Total costs can be divided into a fixed component and a component that is variable with respect to the level of output. When graphed, total costs curve upward.

Total costs can be divided into a fixed component and a component that is variable with respect to the level of output.

A supplier offers to make Part A for $30. Altec Services Corporation has relevant costs of $47 a unit to manufacture 1,020 units of Part A. If there is excess capacity, the opportunity cost of buying Part A from the supplier is

$0 due to excess capacity

Hanung Corp has two service departments, Maintenance and Personnel. Maintenance Department costs of $300,000 are allocated on the basis of budgeted maintenance-hours. Personnel Department costs of $170,000 are allocated based on the number of employees. The costs of operating departments A and B are $188,000 and $282,000, respectively. Data on budgeted maintenance-hours and number of employees are as follows Support Departments Departments ​ Maintenance Department Personnel Department A B Budgeted costs $300,000​ $170,000​ $188,000​ $282,000​ Budgeted maintenance-hours NA ​ 860 ​ 1280 ​ 600 ​ Number of employees 100​ NA 250​ 600​Using the direct method, what amount of Personnel Department costs will be allocated to Department B? (Do not round any intermediary calculations.)

(170000*600)/850=120000

Which of the following is the correct formula for the materials price variance? (Actual price of input - Budgeted price of input) x Actual quantity of input (Actual price of input - Budgeted price of input) x Budgeted quantity of input (Actual quantity of input used - Budgeted quantity of input allowed for actual output) x Budgeted price of input (Actual quantity of input used - Budgeted quantity of input allowed for actual output) x Actual price of input

(Actual price of input - Budgeted price of input) x Actual quantity of input

Which of the following is the correct mathematical expression is used to calculate variable overhead efficiency variance? (Actual quantity × Budgeted rate) - (Budgeted input quantity allowed for actual output × Budgeted rate) (Actual rate − Budgeted rate) × Budgeted quantity (Actual quantity ÷ Budgeted rate) × Budgeted quantity allowed for actual output (Actual quantity ÷ Budgeted rate) − (Budgeted quantity ÷ Budgeted rate)

(Actual quantity × Budgeted rate) - (Budgeted input quantity allowed for actual output × Budgeted rate)

For a manufacturing company, indirect manufacturing costs would be included in ________. merchandise inventory only both work-in-process inventory and finished goods inventory direct materials inventory only direct materials inventory, work-in-process inventory, and finished goods inventory accounts

...

Which of the following ordering of the levels best depicts the cost hierarchy within an ABC system? facility-sustaining level, output unit-level, batch-level, product-sustaining level batch-level, output unit-level, product-sustaining level, and facility-sustaining level output unit-level, batch-level, product-sustaining level, and facility-sustaining level batch-level, output unit-level, facility-sustaining levels, product-sustaining levels (wrong)

...

A company is using a standard cost system and receives its electricity bill. Electricity is considered a variable cost of operations for this company. The bill is for $15,000 and will be paid next month. Which of the following entries would be the correct recording of the electricity bill? Work-in-Process Control $15,000 Accounts Payable $15,000 Work-in-Process Control $15,000 Variable Overhead Allocated $15,000 Variable Overhead Control $15,000 Accounts payable $15,000 Variable Overhead Control $15,000 Variable Overhead Allocated $15,000

...Variable Overhead Control $15,000 Accounts payable $15,000

A.C. Tech Manufacturing Appliances manufactures three sizes of kitchen appliances: small, medium, and large. Product information is provided below. Small Medium Large Unit selling price $450​ $620​ $1,240 Unit costs: Variable manufacturing (240) (280) (530) Fixed manufacturing (80) (140) (250) Fixed selling and administrative (110) (125) (130) Unit profit $20​ $75​ $330 Demand in units 110​ 140​ 110 Machine-hours per unit 40​ 70​ 110 The maximum machine-hours available are 6,500 per week. What is the contribution margin per machine-hour for a medium appliance?

620-280=340/70=4.86

Crandle Manufacturers Inc. is approached by a potential new customer to fulfill a one-time-only special order for a product similar to one offered to domestic customers. The company has excess capacity. The following per unit data apply for sales to regular customers: Variable costs: Direct materials $170 Direct labor 90 Manufacturing support 135 Marketing costs 85 Fixed costs: Manufacturing support 145 Marketing costs 75 Total costs 700 Markup (40%) 280 Targeted selling price $980 What is the contribution margin per unit?

980-(170+90+135+85)=500

Osium Company made the following journal entry: Variable Manufacturing Overhead Allocated 250,000 Variable Manufacturing Overhead Efficiency Variance 80,000 Variable Manufacturing Overhead Control 300,000 Variable Manufacturing Overhead Spending Variance 30,000 Which of the following statements is true of the given journal entry? A $80,000 unfavorable efficiency variance was recorded. A $30,000 unfavorable spending variance was recorded. Work-in-Process is currently overstated. Osium overallocated variable manufacturing overhead.

A $80,000 unfavorable efficiency variance was recorded.

Which of the following statements is true of a peanut-butter costing system? A peanut-butter costing system assumes that all costs are variable. In a peanut-butter costing system, costs of activities are used to assign costs to other cost objects such as products or services based on the activities the products or services consume. A peanut-butter costing system broadly averages or spreads the cost of resources uniformly to cost objects. A peanut-butter costing system typically has more-homogeneous indirect cost pools.

A peanut-butter costing system broadly averages or spreads the cost of resources uniformly to cost objects.

Which of the following statements is true of ABC systems? Service companies find great value from ABC because a vast majority of their cost structure is composed of direct costs. ABC systems provide less insight than traditional systems into the management of the indirect costs. ABC systems is valuable for pricing decisions but not for understanding, managing, and reducing costs in government institutions. ABC systems are used by managers for strategic decisions rather than for inventory valuation in merchandising companies.

ABC systems are used by managers for strategic decisions rather than for inventory valuation in merchandising companies.

Which of the following statements is true of costing systems? Single-indirect cost pool systems always result in overcosting of products. ABC systems classify some indirect costs as direct costs helping to improve processes ABC systems always result in overcosting of products. Single-indirect cost pool systems classify some direct costs as indirect costs.

ABC systems classify some indirect costs as direct costs helping to improve processes

________ is a method of inventory costing in which all variable manufacturing costs and all fixed manufacturing costs are included as inventoriable costs. Variable costing Absorption costing Standard costing Mixed costing

Absorption costing

Which of the following statements is true of absorption costing? Absorption costing allocates fixed manufacturing overhead to actual units produced during the period. Absorption costing shows the same level of profit as variable costing irrespective of the level of inventories. Absorption costing carries over nonmanufacturing costs to the future periods. Absorption costing allocates total manufacturing cost using the budgeted level of production for a particular year.

Absorption costing allocates fixed manufacturing overhead to actual units produced during the period.

Which of the following statements about actual costing is true? Actual costing uses budgeted indirect-cost rates calculated annually. Actual costing uses actual indirect-cost rates calculated annually. Corrective actions can be implemented sooner under actual costing. Manufacturing costs of a job are available earlier under actual costing.

Actual costing uses actual indirect-cost rates calculated annually.

All of the following are possible causes of actual machine hours exceeding budgeted machine hours except: Poor scheduling Machines were not maintained in good operating condition Actual leasing costs for the machine were higher than expected Budgeted standards were set to tight

Actual leasing costs for the machine were higher than expected

Which of the following statements is true? All fixed costs are indirect costs. All direct costs are variable costs. A direct cost of one cost object will always be a direct cost of another cost object. Because of a cost-benefit tradeoff, some direct costs may be treated as indirect costs.

Because of a cost-benefit tradeoff, some direct costs may be treated as indirect costs.

Place the following steps in the order suggested by the seven steps used to assign costs to individual jobs: A. Identify indirect costs B. Compute the total cost of the job C. Select cost-allocation bases D. Compute the indirect cost rate ACDB BACD DCAB CADB

CADB

Which of the following is not true about the cause-and-effect criterion when estimating a cost function? Knowledge of operations can help managers discover cause-and-effect relationships Managers must be careful not to equate high correlation between two variables to mean that either variable causes the other Contractual arrangements may show a direct cause and effect between two variables Correlation of variables proves cause-and-effect

Correlation of variables proves cause-and-effect

Which of the following formulas determine cost of goods sold in a manufacturing entity? Beginning work-in-process inventory + Cost of goods manufactured - Ending work-in-process inventory = Cost of goods sold Cost of goods manufactured - Beginning finished goods inventory - Ending finished goods inventory = Cost of goods sold Beginning work-in-process inventory + Cost of goods manufactured + Ending work-in-process inventory = Cost of goods sold Cost of goods manufactured + Beginning finished goods inventory - Ending finished goods inventory = Cost of goods sold

Cost of goods manufactured + Beginning finished goods inventory - Ending finished goods inventory = Cost of goods sold

Which of the following would not lead to a build up inventory as a strategy to increase operating income? Plant manager switching to make products that absorb the highest amounts of fixed cists Plant manager accepting an order to increase production and build up inventory when another plant in the same company would be better suited to take on the order Cutting overhead costs as year-end approaches. Allocating resources to production by deferring maintenance of equipment and building beyond the current period

Cutting overhead costs as year-end approaches.

Payment of the factory rent would require debits and credits to which accounts? Debit: Factory Depreciation account Credit: Accumulated Depreciation Control Debit: Manufacturing Overhead Control account Credit: Cash Debit: Cost of Goods Sold account Credit: Prepaid Rent Debit: Work-in-Process Control account Credit: Cash

Debit: Manufacturing Overhead Control account Credit: Cash

Which of the following statements about normal costing is true? Direct costs and indirect costs are traced using an actual rate. Direct costs and indirect costs are traced using budgeted rates. Direct costs are traced using a budgeted rate, and indirect costs are allocated using an actual rate. Direct costs are traced using an actual rate, and indirect costs are allocated using a budgeted rate.

Direct costs are traced using an actual rate, and indirect costs are allocated using a budgeted rate.

________ is the continuing reduction in the demand for a company's products that occurs when competitor prices are not met. Downward demand spiral Super-demand cutback Competitor pricing pressure Continuous step down demand

Downward demand spiral

Which of the following is a true statement of energy costs? Strategies to reduce energy costs will not impact variable cost budgets. Energy costs are a fixed cost of doing business for a manufacturer. Energy costs are not controllable Energy costs are a growing component of variable overhead costs.

Energy costs are a growing component of variable overhead costs.

________ costs support the organization as a whole. Unit-level Batch-level Facility-sustaining Product-sustaining

Facility-sustaining

When a job is complete ________. actual indirect manufacturing labor is excluded from the total cost of the job the cost of the job is transferred to Manufacturing Overhead Control it is reduced from Manufacturing Overhead Control account Finished Goods Control is debited

Finished Goods Control is debited

Fixed overhead costs for March for a factory were Salaries of $44,000, depreciation of $10,000, and property taxes of $4,000. Which of the following journal entries would be correct? Fixed Overhead Control $58,000 Accounts Payable $4,000 Salaries Payable $44,000 Accumulated Depreciation $10,000 Work-in-Process $58,000 Accounts Payable $4,000 Salaries Payable $44,000 Accumulated Depreciation $10,000 Depreciation Expense $10,000​ Salaries Expense $44,000 Fixed Overhead Control $4,000 Accumulated Depreciation $10,000 Cash $44,000 Accounts Payable $4,000 Fixed Overhead Control $58,000 Accounts Payable $58,000

Fixed Overhead Control $58,000 Accounts Payable $4,000 Salaries Payable $44,000 Accumulated Depreciation $10,000

Data collection problems arise when ________. fixed and variable costs are not separately identified and both are allocated to products on a per unit basis (i thought this was right?) data are recorded electronically rather than manually accrual-basis costs are used rather than cash-basis costs purely inflationary price effects are removed

Fixed and variable costs are not separately identified

Why is choosing the correct cost driver to estimate indirect manufacturing costs important? Identifying the wrong driver can lead management to incorrect and costly decisions Identifying the wrong driver can increase the usage of the correct driver Identifying the wrong driver can increase actual overhead costs Identifying the wrong driver can increase the direct costs of the cost object

Identifying the wrong driver can lead management to incorrect and costly decisions

Which of the following statements about the direct/indirect cost classification is true? The direct/indirect classification depends on the cost control measures. Indirect costs are always traced. Indirect costs are always allocated. The design of sales target affects the direct/indirect classification.

Indirect costs are always allocated.

Which of the following steps can a management take to reduce the undesirable effects of absorption costing? It can delegate powers to managers to decide which orders they want to accept so that any order which will lead to inventory build-up can be rejected. It can encourage using nonfinancial measures such as units in ending inventory compared to units in sales. It can empower managers to decide the timings of maintenance of plants thereby ensuring that the production is not affected. It can evaluate managers on quarterly basis rather than the usual yearly period thereby mitigating the undesirable effects of absorption costing.

It can encourage using nonfinancial measures such as units in ending inventory compared to units in sales.

Which of the following steps can a management take to reduce the undesirable effects of absorption costing? It can empower managers to decide the timings of maintenance of plants thereby ensuring that the production is not affected. It can delegate powers to managers to decide which orders they want to accept so that any order which will lead to inventory build-up can be rejected. It can evaluate managers on quarterly basis rather than the usual yearly period thereby mitigating the undesirable effects of absorption costing. It can encourage using nonfinancial measures such as units in ending inventory compared to units in sales.

It can encourage using nonfinancial measures such as units in ending inventory compared to units in sales.

Which of the following is a reason for companies to use absorption costing for internal accounting? It does not take into account fixed manufacturing overhead while valuing inventory and hence is more suited for decision making. It is the required inventory method for internal accounting as per GAAP. It measures the cost of all resources, whether manufacturing or nonmanufacturing, necessary to produce inventory. It can help prevent managers from taking actions that make their performance measure look good but that hurt the income they report to shareholders.

It can help prevent managers from taking actions that make their performance measure look good but that hurt the income they report to shareholders.

Which of the following statements is true about analyzing a single variance? It can lead to different other variances. It should be overemphasized to take proper decision. It should be used for quality evaluation. It should be evaluated in isolation from other variances.

It can lead to different other variances.

Which of the following is true of normal capacity utilization? It is also called master-budget capacity utilization It results in the lowest cost estimate of the four capacity options when used for product costing. It will almost always show results that are very close to that of practical capacity utilization It can result in setting selling prices that are not competitive.

It can result in setting selling prices that are not competitive.

A company has a policy "investigate all variances exceeding $3,000 or 15% of the budgeted cost, whichever is lower." There is a variance of $2,000 in repair and maintenance costs of $12,000. What does the company do in the given situation? It should be considered an in-control occurrence. It should be investigated as all variances are equally important. It deserves more attention as it is more than 15% of total repair cost. It should be ignored as it is less than $3,000.

It deserves more attention as it is more than 15% of total repair cost.

Which of the following is a limitation of AARR method It does not track initial investment. It is difficult to compare projects as its result is expressed in dollars and not in percentage terms. It does not consider time value of money. It does not consider income earned throughout a project's expected useful life.

It does not consider time value of money.

Which of the following is true of absorption costing? It expenses marketing costs as cost of goods sold. It treats indirect manufacturing costs as a period cost. It includes fixed manufacturing overhead as an inventoriable cost. It treats direct manufacturing costs as a period cost.

It includes fixed manufacturing overhead as an inventoriable cost.

Which of the following is true of plant utility costs? It is a direct cost. It increases the Materials Control account. It increases the Work-in-Process Control account. It increases the Manufacturing Overhead Control account.

It increases the Manufacturing Overhead Control account.

The gross-margin format is used for ________. mixed costing income statement absorption costing income statement standard costing income statement variable costing income statement

absorption costing income statement

The amount reported for fixed overhead on the static budget is also reported ________. as flexible budget costs as allocated fixed overhead costs as committed variable costs as actual fixed costs

as flexible budget costs

The conference method estimates cost functions ________. using quantitative methods that can be very time consuming and costly based on analysis and opinions gathered from various departments by mathematically analyzing the relationship between inputs and outputs in physical terms using time-and-motion studies

based on analysis and opinions gathered from various departments

The flexible budget contains ________. static budget amounts for planned output budgeted amounts for actual output actual costs for actual output actual costs for planned output

budgeted amounts for actual output

Which of the following is the correct mathematical expression to calculate the fixed overhead production-volume variance? actual costs incurred − fixed overhead allocated for actual output flexible-budget amount − actual costs incurred budgeted fixed overhead − fixed overhead allocated for actual output static-budget amount − flexible-budget amount

budgeted fixed overhead − fixed overhead allocated for actual output

The margin of safety is the difference between ________. budgeted expenses and breakeven expenses budgeted revenues and breakeven revenues actual operating income and budgeted operating income actual sales margin and budgeted sales margin

budgeted revenues and breakeven revenues

The high-low method ________. measures the difference between actual cost and estimated cost for each observation of the cost driver calculates the slope coefficient using only two observed values within the relevant range and their respective costs calculates the standard deviation of residuals measures how well the predicted values, y, based on the cost driver, X, match actual cost observations, Y

calculates the slope coefficient using only two observed values within the relevant range and their respective costs

A well-designed, activity-based cost system helps managers make better decisions because information derived from an ABC analysis ________. is easy to analyze and interpret takes the choices and judgment challenges away from the managers emphasizes how managers can achieve higher sales can be used to eliminate nonvalue-added activities

can be used to eliminate nonvalue-added activities

Service companies and not-for-profit organizations ________. can use CVP by treating all costs as variable cannot use CVP because they don't manufacture a product can use CVP by focusing on measuring the organization's output cannout use CVP because there is no way to distinguish fixed and variable costs

can use CVP by focusing on measuring the organization's output

Which of the following best defines standard costing? It is a system that allocates overhead costs on the basis of standard overhead cost rates times the actual quantities of the allocation based used. It is a system that traces direct cost to output by multiplying actual process or rates by actual quantities of inputs + allocates overhead by on the basis of actual quantities of the allocation base used. It is a system that traces direct costs to output produced by multiplying the standard prices or rates by the standard quantities of inputs allowed for the actual output produced. It is the same as actual costing but done in real time.

It is a system that traces direct costs to output produced by multiplying the standard prices or rates by the standard quantities of inputs allowed for the actual output produced.

Which of the following statements is true of benchmarking? It fails to help to improve organizational performance as benchmarking data does not provide insight into why costs or revenues differ across companies. It is a systematic approach of optimizing business processes. It is difficult to ensure that the benchmark numbers are comparable due to the existence of differences across companies. It considers four major business aspects such as financial, customer, internal business processes, and learning and growth

It is difficult to ensure that the benchmark numbers are comparable due to the existence of differences across companies.

Which of the following is not one of the reasons why absorption costing might also be used for internal reporting? For long-term decision making both variable and fixed costs must be considered for inventory related decisions It is cost effective and less confusing for managers to use one common method for both internal and external reporting It can help prevent managers from making decisions that make their performance look good to the detriment of income reported to shareholders It is more useful for managerial decision making than variable costing

It is more useful for managerial decision making than variable costing

The production-volume variance may also be referred to as the ________. efficiency variance flexible-budget variance spending variance denominator-level variance

denominator-level variance

Which of the following best describes practical capacity? It is the level of capacity utilization that satisfies average customer demand over a period that includes seasonal, cyclical, and trend factors. It is the level of capacity utilization that managers expect for the current budget period, which is typically one year. It is the level of capacity that reduces theoretical capacity by considering unavoidable operating interruptions, such as scheduled maintenance time and shutdowns for holidays. It is the level of capacity based on producing at full efficiency all the time.

It is the level of capacity that reduces theoretical capacity by considering unavoidable operating interruptions, such as scheduled maintenance time and shutdowns for holidays.

Which of the following statements is true of a linear cost function? It presents total cost as slope coefficient. It presents variable cost as a slope coefficient. It presents variable cost as an intercept. It presents total cost as an intercept.

It presents variable cost as a slope coefficient

Which of the following statements is true of a post-investment audit? It helps managers avoid optimistic estimate errors. It provides managers with feedback about the performance of a project to determine if any variance from expectations were the result of the overly optimistic estimates of because of implementation issues. It does not help senior management to recognize problems in the implementation of the project. It encourages managers to overstate the expected cash inflows from projects and accept projects they should reject.

It provides managers with feedback about the performance of a project to determine if any variance from expectations were the result of the overly optimistic estimates of because of implementation issues.

Which of the following is a reason for companies adopting variable costing for internal reporting purposes? It measures the cost of all manufacturing resources, whether variable or fixed, necessary to produce inventory. It is cost-effective to use variable costing for both internal and external reporting. It reduces the incentives for undesirable buildup of inventories. It assists in accurate pricing decisions in case of long-run pricing.

It reduces the incentives for undesirable buildup of inventories.

Which of the following is true of refinement of a costing system? It is likely to yield the most decision-making benefits when direct costs are a high percentage of total costs. A homogeneous cost pool will use multiple cost drivers to allocate costs. It reduces the use of broad averages for assigning the cost of resources to cost objects. While refining a costing system, companies should identify as many indirect costs as is economically feasible.

It reduces the use of broad averages for assigning the cost of resources to cost objects.

Which of the following is true of variable costing? It includes fixed manufacturing overhead as an inventoriable cost. It is required for external reporting to shareholders. It expenses administrative costs as cost of goods sold. It treats direct manufacturing costs as a product cost.

It treats direct manufacturing costs as a product cost.

Which of the following is true of the Work-in-Process Control account? It is an expense account. It tracks overhead costs in-process from beginning through completion. Its balance is the sum of amounts from all in-process individual job-cost records. It tracks all direct material purchases.

Its balance is the sum of amounts from all in-process individual job-cost records.

Which of the following is true of the Work-in-Process Control account? It tracks all direct material purchases. It is an expense account. It tracks overhead costs in-process from beginning through completion. Its balance is the sum of amounts from all in-process individual job-cost records.

Its balance is the sum of amounts from all in-process individual job-cost records.

Southwestern College is planning to hold a fund raising banquet at one of the local country clubs. It has two options for the banquet: OPTION one: Crestview Country Club a. Fixed rental cost of $1,000 b. $12 per person for food OPTION two: Tallgrass Country Club a. Fixed rental cost of $3,000 b. $8.00 per person for food Southwestern College has budgeted $1,800 for administrative and marketing expenses. It plans to hire a band which will cost another $800. Tickets are expected to be $30 per person. Local business supporters will donate any other items required for the event. Which option provides the least amount of risk? Option one Option two Both options provide the same amount of risk. It depends on how many donations it receives.

Option one

Kanga Company is considering two different production plans. Option one: Fixed costs of $10,000 and a breakeven point of 500 units. Option two: Fixed costs of $20,000 and a breakeven point of 700 units. Which option should Kanga choose if it is expecting to produce 600 units? Option two. Option one. Both options are equally good. It isn't possible to determine from the information given.

Option one.

In planning and control of capacity costs, managers must consider possible capacity measures. Which of the following measures the available supply of capacity in a factory? Master-budget capacity Normal capacity Practical capacity Theoretical capacity

Practical capacity

Which of the following assumes that capacity will be decreased because of slowdowns due to plant maintenance or other interruptions of the production lines? Theoretical capacity Practical capacity Master-budget capacity utilization Normal capacity utilization

Practical capacity

Which one of the following items is a direct cost? The salary of a maintenance supervisor in a multiproduct manufacturing plant; Product B is the cost object. Printing costs incurred for payroll check processing; payroll check processing is the cost object. Utility costs of the administrative offices; the accounting department is the cost object. Customer-service costs of a multiproduct firm; Product A is the cost object.

Printing costs incurred for payroll check processing; payroll check processing is the cost object.

Yellow Mountain Manufacturing factors practical capacity as a denominator to calculate budgeted fixed overhead. Theoretical capacity is 12,000 units per year with practical capacity of 9,000 units per year. Budgeted fixed overhead costs were $690,000 and actual overhead costs were $730,000 with actual output of 8,000 units. Which of the following statements is true? The budgeted cost per unit of supplying the capacity was $86.25 The budgeted cost of supplying the capacity was $57.50 per unit. The actual cost of supplying capacity was $76.67 per unit The budgeted cost of supplying the capacity was $76.67 per unit

The budgeted cost of supplying the capacity was $76.67 per unit

When using the high-low method, the numerator of the equation that determines the slope is the ________. difference between the high and low observations of the cost driver difference between the fixed cost and variable cost associated with the cost driver difference between the costs associated with highest and lowest observations of the cost driver difference between the positive and negative values of dependent and independent variables

difference between the costs associated with highest and lowest observations of the cost driver

Which of the following statements is true of fixed overhead cost variances? The difference between actual costs and flexible budget costs will give the production volume variance. The difference between flexible budget costs and allocated overhead costs will give the production volume variance. The difference between static budget costs and flexible budget costs will give the production volume variance. The difference between actual costs and static budget costs will give the production volume variance.

The difference between flexible budget costs and allocated overhead costs will give the production volume variance.

Which of the following is true with activity based cost accounting? Activity-based costing is more likely to result in major differences from traditional costing systems if the firm manufactures only one product rather than multiple products. Chances of product-cost cross-subsidization are higher in activity-based costing compared to traditional costing systems. The focus is on activities that account for a sizable fraction of indirect costs . Activity-based costing ignores the allocation of marketing and distribution costs.

The focus is on activities that account for a sizable fraction of indirect costs .

Which of the following is true of cost-volume-profit analysis? The theory states that total variable costs remain the same over a relevant range. The theory assumes that all costs are variable. The theory assumes that units manufactured equal units sold. The theory states that total costs remain the same over the relevant range.

The theory assumes that units manufactured equal units sold.

Which of the following is true of cost-volume-profit analysis? The theory states that total variable costs remain the same over a relevant range. The theory states that total costs remain the same over the relevant range. The theory assumes that units manufactured equal units sold. The theory assumes that all costs are variable.

The theory assumes that units manufactured equal units sold.

In the graph method of CVP analysis

The total revenue line starts at the origin and the total costs line starts at the fixed intercept.

Product-sustaining costs in activity-based costing are similar to

fixed costs

Which of the following is a direct manufacturing cost? fringe benefits paid to machine operators plant rent plant maintenance property taxes on plant

fringe benefits paid to machine operators

A plant manufactures several different products. If a particular product is the cost object, then the wages of the plant supervisor can be classified as a

indirect manufacturing cost and therefore is an inventoriable cost

Which of the following has accelerated need for refined cost systems? a shift toward increased direct costs intense competition global monopolies rising prices

intense competition

Product costs for financial statements are

inventoriable costs for external reporting purposes

The cost function y = 2,700 + 8X

is a straight line

The marketing manager's performance evaluation is most fair when based on a denominator level using ________ as it is the principal short-run planning and control tool. theoretical capacity normal capacity utilization master-budget capacity utilization practical capacity

master-budget capacity utilization

Which of the following is a reason that has accelerated the demand for refinements to the costing system? The increasing competition in product markets has led to an increase in contribution margin resulting in a decrease of break even point. The increased of automated processes has led to the increase in direct manufacturing cost leading to a decrease in break even point. The declining demand for customized products has led managers to decrease the variety of products and services their companies offer. The use of product and process technology has led to an increase in indirect costs and a decrease in direct costs.

The use of product and process technology has led to an increase in indirect costs and a decrease in direct costs.

________ is the level of capacity based on producing at full efficiency all the time. Practical capacity Normal capacity Theoretical capacity Demand capacity

Theoretical capacity

Which of the following best describes how fixed cost are treated in a variable cost method? They are classified as nonmanufacturing costs They are allocated to the product cost using a denominator-level capacity choice They are excluded from inventory cost and are treated as period costs They are part of the product cost

They are excluded from inventory cost and are treated as period costs

Which of the following is a disadvantage of using the standards developed by a firm itself to develop a budget? The expected future changes are not included in the standards. A firm's inefficiencies will be part of the data. They are not based on realized benchmarks and can be unrealistic The flexible-budget amounts are difficult to determine.

They are not based on realized benchmarks and can be unrealistic

Atlas Manufacturing produces a unique valve, and has the capacity to produce 50,000 valves annually. Currently Atlas produces 40,000 valves and is thinking about increasing production to 45,000 valves next year. What is the most likely behavior of total manufacturing costs and unit manufacturing costs given this change? Total manufacturing costs will increase and unit manufacturing costs will decrease. Total manufacturing costs will increase and unit manufacturing costs will also increase. Total manufacturing costs will increase and unit manufacturing costs will stay the same. Total manufacturing costs will stay the same and unit manufacturing costs will stay the same.

Total manufacturing costs will increase and unit manufacturing costs will decrease.

Regression analysis ________. measures the average amount of change in the dependent variable associated with a unit change in one or more independent variables measures the variability or dispersion in a set of data points estimates the cost functions using the time-and-motion studies calculates the slope coefficient using only two observed values within the relevant range and their respective costs

measures the average amount of change in the dependent variable associated with a unit change in one or more independent variables

Which of the following is a fixed cost with respect to units produced in a factory? electricity expenses utilities cost of the building monthly rent payment for the building direct material costs

monthly rent payment for the building

The balance sheet of a service-sector companies would show ________. work-in-process inventory, and finished goods inventory accounts direct materials inventory, work-in-process inventory, and finished goods inventory accounts only finished goods inventory no inventory accounts

no inventory accounts

It is most difficult to estimate ________ because of the need to predict demand for the next few years. master-budget capacity utilization practical capacity theoretical capacity normal capacity utilization

normal capacity utilization

Under variable costing, if a manager's bonus is tied to operating income, then increasing inventory levels compared to last year would result in ________. being unable to determine the manager's bonus using only the above information not affecting the manager's bonus increasing the manager's bonus decreasing the manager's bonus

not affecting the manager's bonus

For a business that offers customers a store where product can be purchased and picked up or a delivery service that can ship the product directly to the customer, which of the following would most likely be the best cost allocation base for distribution costs? number of pounds of product shipped or delivered number of customer service phone calls and emails per period Electricity costs for the period Number of products sold

number of pounds of product shipped or delivered

Which of the following capacity levels do proponents of activity-based costing recommend to be used as the denominator level to calculate activity cost rates? master-budget capacity utilization practical capacity normal capacity utilization theoretical capacity

practical capacity

A purchasing manager's performance is best evaluated using information such as price and terms bargaining effectiveness, achievement of quality goals, and direct materials price variance direct manufacturing labor flexible-budget variance direct materials flexible-budget variance usage efficiency and direct materials price variance

price and terms bargaining effectiveness, achievement of quality goals, and direct materials price variance

The flexible-budget variance for direct cost inputs can be further subdivided into a ________. static-budget variance and a price variance static-budget variance and a sales-volume variance price variance and an efficiency variance sales-volume variance and an efficiency variance

price variance and an efficiency variance

Lavender Company is a logistics company and has recently implemented ABC system. Using activity-based information, it decides to reduce the bulkiness of the packages delivered, thereby reducing costs. This suggests that ABC system helps managers in ________ decisions. pricing product-design product-mix process-improvement

process-improvement

If a sales-volume variance was caused by poor-quality products, then the ________ would be in the best position to explain the variance. sales supervisor production manager financial supervisor logistic manager

production manager

If a sales-volume variance was caused by poor-quality products, then the ________ would be in the best position to explain the variance. financial supervisor logistic manager sales supervisor production manager

production manager

Practical capacity is the denominator-level concept that ________. is based on anticipated levels of capacity utilization for the coming budget period is the maximum level of operations at maximum efficiency is based on the level of capacity utilization that satisfies average customer demand over periods generally longer than one year reduces theoretical capacity for unavoidable operating interruptions

reduces theoretical capacity for unavoidable operating interruptions

The cost function y = 13,000 + 9X ________. has a slope coefficient of 13,000 is a curved line will intersect the y-axis at 9 represents a mixed cost

represents a mixed cost

Contribution margin equals ________. revenues minus period costs revenues minus fixed costs revenues minus product costs revenues minus variable costs

revenues minus variable costs

One possible means of determining the difference between operating incomes for absorption costing and variable costing is by ________. adding fixed manufacturing costs to the production-volume variance subtracting sales of the previous period from sales of this period multiplying the number of units produced by the budgeted fixed manufacturing cost rate subtracting fixed manufacturing overhead in beginning inventory from fixed manufacturing overhead in ending inventory

subtracting fixed manufacturing overhead in beginning inventory from fixed manufacturing overhead in ending inventory

Ways to "produce for inventory" that result in increasing operating income include ________. switching production to products that absorb the most amounts of fixed manufacturing costs switching production to products that absorb the least amounts of fixed manufacturing costs undervaluing ending inventory by not recording certain costs that have been incurred delaying items that absorb the greatest amount of fixed manufacturing costs

switching production to products that absorb the most amounts of fixed manufacturing costs

A learning curve is a function ________. where unit costs increase as productivity increases that is linear that measures the decline in labor-hours per unit due to workers becoming better at a job that increases at a greater rate as workers become more familiar with their tasks

that measures the decline in labor-hours per unit due to workers becoming better at a job

Which of the following is true of CVP analysis? Proportion of different products will vary according to demand and supply when multiple products are sold. Total revenues and total costs are linear in relation to output units. Unit selling price, unit variable costs, and unit fixed costs are known and remain constant. Costs may be separated into separate inventoriable and period components with respect to the level of output.

Total revenues and total costs are linear in relation to output units.

When variances are immaterial, which of the following statements is true of the journal entry to write-off the variable overhead variance accounts? Cost of Goods Sold account will always be debited. Unfavorable efficiency variance will be credited. Cost of Goods Sold account will always be credited. Favorable efficiency variance will be credited.

Unfavorable efficiency variance will be credited.

Marshall Company uses a standard cost system. In March, $270,000 of variable manufacturing overhead costs were incurred and the flexible-budget amount for the month was $310,000. Which of the following variable manufacturing overhead entries would have been recorded for March? Variable Manufacturing Overhead Allocated 310,000 Accounts Payable and other accounts 310,000 Work-in-Process Control 270,000 Accounts Payable Control and other accounts 270,000 Accounts Payable Control and other accounts 310,000 Work-in-Process Control 310,000 Variable Manufacturing Overhead Control 270,000 Accounts Payable Control and other accounts 270,000

Variable Manufacturing Overhead Control 270,000 Accounts Payable Control and other accounts 270,000

Which of the following statements is true of variable overhead costs? Variable overhead costs have no production-volume variance. Variable overhead costs have no spending variance. Variable overhead costs always have unused capacity. Variable overhead costs have no efficiency variance.

Variable overhead costs have no production-volume variance.

A plot of cost driver data and cost data may show all but the following: a positive relationship cause-and-effect relationship a strong relationship linear relationship

cause-and-effect relationship

Demand for refinements to the costing system has accelerated due to ________. increase in direct costs decrease in indirect costs competition in product markets decrease in product diversity

competition in product markets

An unfavorable sales-volume variance could result from ________. competitors taking market share an inefficiency of a purchasing manager in bargaining with suppliers a decrease in actual selling price compared to anticipated selling price an inappropriate assignment of labor or machines to specific jobs

competitors taking market share

A single indirect-cost rate distorts product costs because ________. it assumes all costs are product costs competitive pricing is ignored it recognizes specific activities that are required to produce a product there is an assumption that all support activities affect all products in a uniform way

there is an assumption that all support activities affect all products in a uniform way

Variance analysis should be used ________. as the sole source of information for performance evaluation to set the standards which are very easy to achieve to encourage employees to focus on meeting standards to understand why variances arise and to improve future performance to punish employees that do not meet standards

to understand why variances arise and to improve future performance

A major concern that arises with multiple regression is multicollinearity, which exists when ________. two or more independent variables are highly correlated with one another the R2 statistic is low the dependent variable is not normally distributed the standard errors of the coefficients of the individual variables decrease

two or more independent variables are highly correlated with one another

Actual costing is a costing system that traces direct costs to a cost object by ________. using the budgeted direct cost rates times the budgeted quantities of direct-cost inputs using the budgeted direct cost rates times the actual quantities of the direct cost inputs using the actual direct cost rates times the actual quantities of the direct-cost inputs using the actual direct costs rates times the budgeted quantities of the direct-cost inputs

using the actual direct cost rates times the actual quantities of the direct-cost inputs

Crimson Services, Inc., employs 8 individuals. They are all paid $16.50 per hour. How would total costs of personnel be classified? variable cost irrelevant cost mixed cost fixed cost

variable cost

The contribution-margin format is used for ________. absorption costing income statement mixed costing income statement job order costing income statement variable costing income statement

variable costing income statement

Assume only the specified parameters change in a CVP analysis. The contribution margin percentage increases when ________. variable costs per unit increase total fixed costs decrease variable costs per unit decrease total fixed costs increase

variable costs per unit decrease

inventoriable costs are expensed on the income statement ________. when direct materials for the product are purchased when the direct materials are used in production. when the products are sold when the goods move from work-in process to finished goods account

when the products are sold

In CVP analysis, focusing on target net income rather than operating income ________. will help managers construct a better capital policy will not change the breakeven point will decrease the breakeven point will increase the breakeven point

will not change the breakeven point

Which of the following is not reported on the income statement of a manufacturing firm? administrative and selling expenses marketing and distribution costs cost of goods sold work in progress

work in progress

An unfavorable efficiency variance for direct manufacturing labor might indicate that ________. there is unexpected increase in direct labor rates more higher-skilled workers were scheduled than planned work is scheduled inefficiently lower-quality materials were purchased

work is scheduled inefficiently

Capital budgeting is both a decision making and control tool. Which of the following is an example of capital budgeting as a control tool? A company uses capital budgeting techniques to evaluate a group of prospective alternative projects. A large manufacturer sets up a "capital relief" fund to help supplement sustainability projects that would not meet targeted rates of returns without the capital relief fund assistance. A company's capital project is not meeting the level of profitability expected, will not meet the targeted NPV, and is abandoned. When considering capital expenditures, a company looks at a minimum of six potential (alternative) projects.

When considering capital expenditures, a company looks at a minimum of six potential (alternative) projects

Which of the following journal entries is used to record fixed overhead costs allocated? Work-in-Process Control Fixed Overhead Allocated Fixed Overhead Allocated Work-in-Process Control Fixed Overhead Allocated Fixed Overhead Control Fixed Overhead Control Work-in-Process Control

Work-in-Process Control Fixed Overhead Allocated

Teddy Company uses a standard cost system. In May, $234,000 of variable manufacturing overhead costs were incurred and the flexible-budget amount for the month was $240,000. Which of the following variable manufacturing overhead entries would have been recorded for May? Work-in-Process Control 240,000 Variable Manufacturing Overhead Allocated 240,000 Work-in-Process Control 234,000 Accounts Payable Control and other accounts 234,000 Accounts Payable Control and other accounts 234,000 Variable Manufacturing Overhead Control 234,000 Accounts Payable Control and other accounts 240,000 Work-in-Process Control 240,000

Work-in-Process Control 240,000 Variable Manufacturing Overhead Allocated 240,000

Which of the following general ledger accounts will have a subsidiary ledger account? Work-in-Process Control account Joe's Accounts Receivable subsidiary account Operating Expenses account Cost of Goods Sold account

Work-in-Process Control account

In a costing system, ________. a cost object should be a product and not a department or a geographic territory cost tracing allocates indirect costs cost allocation assigns direct costs a cost-allocation base can be either financial or nonfinancial

a cost-allocation base can be either financial or nonfinancial

When machine-hours are used as an overhead cost-allocation base, the most likely cause of a favorable variable overhead spending variance is ________. a decline in the cost of energy excessive machine breakdowns strengthened demand for the product the production scheduler efficiently scheduled jobs

a decline in the cost of energy

Which of the following can be a reason for a favorable price variance for direct materials? a decrease in the price of materials due to an oversupply of materials less amount of material used during production than planned for actual output an unexpected increase in the price of materials workers taking less time to produce the products

a decrease in the price of materials due to an oversupply of materials

A job-cost record uses information from ________. a labor-time card to record an employee's wage rate and hours spent on a particular job the bill of materials to ensure the goods are of the prescribed quality a materials-requisition report to record the type and quantity of item received in an order from a supplier a materials-requisition record to record raw material purchases from suppliers

a labor-time card to record an employee's wage rate and hours spent on a particular job

Which of the following is true about what the Internal Revenue Service requires for calculating indirect manufacturing costs per unit? requires the use of master-budget capacity utilization requires that theoretical capacity be used as a means of allocating indirect manufacturing costs a method of which fairly apportions indirect production costs among the various items produced requires the use of normal capacity utilization

a method of which fairly apportions indirect production costs among the various items produced

Which of the following is true about what the Internal Revenue Service requires for calculating indirect manufacturing costs per unit? requires the use of normal capacity utilization requires the use of master-budget capacity utilization requires that theoretical capacity be used as a means of allocating indirect manufacturing costs a method of which fairly apportions indirect production costs among the various items produced

a method of which fairly apportions indirect production costs among the various items produced

Using department indirect-cost rates to allocate costs will result in results similar to ABC if: several activities cause a sizable portion of the costs of the department a single activity accounts for a sizable portion of the costs of the department a single activity accounts for a small portion of the costs of the department several activities cause a small portion of the costs of the department

a single activity accounts for a sizable portion of the costs of the department

Sensitivity analysis is ________. a way of seeing how employees will be affected by changes a way of determining what will happen if assumptions change a way of seeing how far from budget actual results are a way of determining how customers will react to new products

a way of determining what will happen if assumptions change

For a company with diverse products, undercosting overhead of a product will lead to product-cross -subsidization which means that: direct labor costs of the product are misallocated direct material costs of the product are misallocated indirect costs of another product are misallocated direct costs of another product are misallocated

indirect costs of another product are misallocated

Which of the following is an example of time-series data: indirect labor costs and machine-hours in a manufacturer's assembly department each month for the last 12 months Store sales for each U.S. based Wal-Mart for each the last 10 years. Direct labor hours in the Boston, Massachusetts and the New York City facility for each of the last 12 months loan processing times in each of 26 similar branch offices over the last 12 months

indirect labor costs and machine-hours in a manufacturer's assembly department each month for the last 12 months

Which of the following is a period cost? costs incurred to provide customer service such as the operation of a 800 phone line to trouble shoot product problems and to answer questions about product warranties the amount of depreciation expense recognized because of the use of plant assets such as building and equipment the cost of materials used to make a product when those materials are an insignificant part of the finished product the cost of direct labor

costs incurred to provide customer service such as the operation of a 800 phone line to trouble shoot product problems and to answer questions about product warranties

Which of the following is an example of time-series data: indirect labor costs and machine-hours in a manufacturer's assembly department each month for the last 12 months Store sales for each U.S. based Wal-Mart for each the last 10 years. Direct labor hours in the Boston, Massachusetts and the New York City facility for each of the last 12 months loan processing times in each of 26 similar branch offices over the last 12 months

indirect labor costs and machine-hours in a manufacturer's assembly department each month for the last 12 months

A job that shows low profitability may be the result of ________. insurance claim of the damaged goods overpricing the job inefficient direct manufacturing labor excessive usage of direct materials

inefficient direct manufacturing labor

The cost allocation base ________. is a grouping of individual indirect cost items is anything for which a measurement of costs is desired are costs related to a particular cost object that cannot be traced to that cost object in an economically feasible way is a systematic way to link an indirect cost or group of indirect costs to cost objects

is a systematic way to link an indirect cost or group of indirect costs to cost objects

Bismith Company reported: Actual fixed overhead $700,000 Fixed manufacturing overhead spending variance $40,000 unfavorable Fixed manufacturing production-volume variance $30,000 unfavorable To record the write-off of these variances at the end of the accounting period, Bismith would ________. debit Fixed Manufacturing Overhead Spending Variance for $40,000 credit Fixed Manufacturing Overhead Allocated for $700,000 debit Fixed Manufacturing Control for $700,000 credit Fixed Manufacturing Production-Volume Variance for $30,000

debit Fixed Manufacturing Overhead Spending Variance for $40,000

In a normal costing system, the Manufacturing Overhead Control account ________. is decreased by allocated manufacturing overhead is credited with amounts transferred to Work-in-Process is debited with actual overhead costs is increased by allocated manufacturing overhead

is debited with actual overhead costs

The following information is for Alex Corp: Product X: Revenue $15.00 Variable Cost $2.50 Product Y: Revenue $25.00 Variable Cost $10.00 Total fixed costs $50,000 If the sales mix shifts to one unit of Product X and two units of Product Y, then the breakeven point will ________. will be greater than the original breakeven point decrease increase stay the same

decrease

A step variable-cost function ________. is often approximated with a continuous variable-cost function example includes adding additional warehouse space remains the same over a wide range of activity is fixed over the long run but not over the short run

is often approximated with a continuous variable-cost function

In the estimation of a cost function using quantitative analysis, the independent variable ________. is the product of fixed costs and slope coefficient is the cost to be predicted is the product of total costs and slope coefficient is the factor used to predict the dependent variable

is the factor used to predict the dependent variable

Which of the following best describes the internal rate-of-return (IRR) method? it calculates the discount rate at which sum of an investment's present value of all expected cash inflows equals the present value of its expected cash outflows. it calculates the discount rate at which an investment's present value of the total of all expected cash inflows equals the present value of its expected cash outflows. (wrong) it calculates the discount rate at which an investment's total of all expected cash inflows equals the present value of its expected cash outflows. it calculates the discount rate at which an investment's future value of all expected cash inflows equals the present value of its expected cash outflows.

it calculates the discount rate at which sum of an investment's present value of all expected cash inflows equals the present value of its expected cash outflows. (i think)

It only makes sense to implement an ABC system when ________. a single product is produced in bulk it traces more costs as direct costs production process is labor-intensive its benefits exceed its implementation costs

its benefits exceed its implementation costs

The basic source document for direct manufacturing labor is the ________. labor-requisition record labor-time record job-cost record materials-requisition record

labor-time record

Which of the following is an example of nonlinear cost function? mixed cost functions fixed-cost functions variable-cost functions learning curves

learning curves

A favorable efficiency variance for direct manufacturing labor indicates that ________. a lower wage rate than planned was paid for direct labor more direct manufacturing labor-hours were used during production than planned for actual output less direct manufacturing labor-hours were used during production than planned for actual output a higher wage rate than planned was paid for direct labor

less direct manufacturing labor-hours were used during production than planned for actual output

The following information is for the Jeffries Corporation: Product A: Revenue $16.00 Variable Cost $12.00 Product B: Revenue $24.00 Variable Cost $16.00 Total fixed costs $75,000 Assume the sales mix consists of three units of Product A and one unit of Product B. If the sales mix shifts to four units of Product A and one unit of Product B, then the weighted-average contribution margin will ________. cannot be determined from this information stay the same decrease per unit increase per unit

decrease per unit

Depreciation is usually NOT considered an operating cash flow in capital budgeting because ________. depreciation usually does not result in an increase in working capital depreciation usually has no effect on the disposal price of the machine deducting depreciation from operating cash flows would be counting the lump-sum amount twice depreciation is usually a constant amount each year over the life of the capital investment

deducting depreciation from operating cash flows would be counting the lump-sum amount twice

The cost to be predicted and managed is referred to as the ________. regression cost driver dependent variable independent variable

dependent variable

A favorable efficiency variance for direct manufacturing labor indicates that ________. less direct manufacturing labor-hours were used during production than planned for actual output a lower wage rate than planned was paid for direct labor more direct manufacturing labor-hours were used during production than planned for actual output a higher wage rate than planned was paid for direct labor

less direct manufacturing labor-hours were used during production than planned for actual output

In a company with low operating leverage, ________. fixed costs are more than the contribution margin less risk is assumed than in a highly leveraged firm there is a higher possibility of net loss than a higher-leveraged firm contribution margin and operating income are inversely related

less risk is assumed than in a highly leveraged firm

Which of the following is a stage of the capital budgeting process that determines which investment yields the greatest benefit and the least cost to an organization? identify projects stage implement the decision, evaluate performance, and learn stage make predictions stage make decisions by choosing among alternatives stage

make decisions by choosing among alternatives stage

Which of the following is a stage of the capital budgeting process that forecasts all potential cash flows attributable to the alternative projects? make predictions stage implement the decision, evaluate performance, and learn stage make decisions by choosing among alternatives stage identify projects stage

make predictions stage

To discourage producing for inventory, management can ________. implement absorption costing across all departments develop budgeting and planning activities that reduce management's freedom to inappropriately build inventory through increased production discourage using nonfinancial measures such as units in ending inventory compared to units in sales as nonfinancial measures may not be congruent with management performance goals evaluate performance over a quarterly period rather than a single year

develop budgeting and planning activities that reduce management's freedom to inappropriately build inventory through increased production

The goal of a properly constructed ABC system is to ________. develop the best cost system that meets the cost/benefit test identify more indirect costs have separate allocation rates for each department have the most accurate cost system

develop the best cost system that meets the cost/benefit test

The goal of a properly constructed ABC system is to ________. develop the best cost system that meets the cost/benefit test have separate allocation rates for each department identify more indirect costs have the most accurate cost system

develop the best cost system that meets the cost/benefit test

For a manufacturing firm that produces multiple families of products requiring various combinations of different types of parts, what would be the best allocation base for human resource adminstration costs? machine hours number of parts purchased electricity costs direct labor hours

direct labor hours

Which of the following increases (are debited to) the Work-in-Process Control account? marketing expenses actual plant insurance costs direct manufacturing labor costs customer services costs

direct manufacturing labor costs

Total manufacturing costs are comprised of ________. direct materials costs, direct manufacturing labor costs, and manufacturing overhead costs indirect materials costs, indirect manufacturing labor costs, and manufacturing overhead costs direct materials costs and period costs prime costs and period costs

direct materials costs, direct manufacturing labor costs, and manufacturing overhead costs

For a manufacturing company, direct material costs may be included in ________. direct materials inventory, work-in-process inventory, and finished goods inventory accounts only the direct materials inventory account only in the merchandise inventory account reflecting the inventory on hand only in both work-in-process inventory and finished goods inventory

direct materials inventory, work-in-process inventory, and finished goods inventory accounts

The balance sheet of a manufacturing-sector companies would report ________. direct materials inventory and finished goods inventory accounts only only merchandise inventory only finished goods inventory direct materials inventory, work-in-process inventory, and finished goods inventory accounts

direct materials inventory, work-in-process inventory, and finished goods inventory accounts

For manufacturing firms, inventoriable costs include ________. plant supervisor salaries advertising costs to promote the sale of finished goods costs of dealing with customer questions about warranties after the sale distribution costs such the cost of operating a fleet of delivery vehicles

distribution costs such the cost of operating a fleet of delivery vehicles (i think)

The planned operating income is calculated by ________. multiplying net income by tax rate dividing net income by 1 − tax rate dividing net income by tax rate multiplying net income by 1 − tax rate

dividing net income by 1 − tax rate

Service-sector companies ________. do not manufacture a product and therefore do not apply or adapt cost accounting to their operations. purchase and then sell tangible products by changing their basic form and therefore need cost accounting. purchase merchandise then sell it without changing its basic form and therefor do not need cost accounting systems. do not produce tangible products but cost accounting is still utilized in service firms.

do not produce tangible products but cost accounting is still utilized in service firms.

For fixed manufacturing overhead, there is no ________. efficiency variance spending variance production-volume variance flexible-budget variance

efficiency variance

Which one of the following is a variable cost for an insurance company? electricity expenses rent of the building CEO's salary property taxes

electricity expenses

Which one of the following is a variable cost for an insurance company? rent of the building electricity expenses CEO's salary property taxes

electricity expenses

Activity-based costing (ABC) can eliminate cost distortions because ABC systems ________. use single cost pool for all overhead costs, thereby enabling simplicity never consider interactions between different departments in assigning support costs use a broad average to allocate all overhead costs establish a cause-and-effect relationship with the activities performed

establish a cause-and-effect relationship with the activities performed

A general rule in capital budgeting is that a project is accepted only if the internal rate of return equals or ________. exceeds the inflation rate exceeds the risk-free rate exceeds the accrual accounting rate of return exceeds the required rate of return

exceeds the required rate of return

The focus in capital budgeting should be on ________. expected future cash flows that differ between alternatives allocation of overheads favorable and unfavorable variance expenses under accrual accounting

expected future cash flows that differ between alternatives

For a company which produce its products in batches, the CEO's salary is a(n) ________ cost. batch-level output unit-level product-sustaining facility-sustaining

facility-sustaining

It is usually difficult to find good cost driver (cause-and-effect relationship) between ________ and a cost allocation base. batch-level costs facility-sustaining costs unit-level costs product-sustaining costs

facility-sustaining costs

Which of the following is true if the production volume decreases? variable cost per unit decreases variable cost per unit increases average cost per unit decreases fixed cost per unit increases

fixed cost per unit increases

Which of the following is true if the production volume decreases? variable cost per unit increases average cost per unit decreases variable cost per unit decreases fixed cost per unit increases

fixed cost per unit increases

In flexible budgets the costs that are not "flexed" because they remain the same within a relevant range of activity (such as sales or output) are called ________. variable costs fixed costs total overhead costs total budgeted costs

fixed costs

Which of the following is a problem related to cost analysis? extreme observations are adjusted or removed fixed costs are allocated as if they are variable costs inflationary effects are removed a company keeps accounting records on the accrual basis

fixed costs are allocated as if they are variable costs

The breakeven point revenues is calculated by dividing ________. total revenues by fixed costs fixed costs by contribution margin percentage fixed costs by total revenues contribution margin percentage by fixed costs

fixed costs by contribution margin percentage

Breakeven point in units is ________. contribution margin per unit divided by total cost per unit total costs divided by profit margin per unit fixed costs divided by contribution margin per unit the sum of fixed and variable costs divided by contribution margin per unit

fixed costs divided by contribution margin per unit

There is no output-level variance for variable costing, when ________. fixed manufacturing overhead is allocated to work in process fixed manufacturing overhead is not allocated to work in process the inventory level increases during the period the inventory level decreases during the period

fixed manufacturing overhead is not allocated to work in process

Under standard costing, ________. fixed overhead costs are treated as if they are a sunk cost fixed overhead costs are treated as if they are a fixed cost variable overhead costs are treated as if they are a fixed cost fixed overhead costs are treated as if they are a variable cost

fixed overhead costs are treated as if they are a variable cost

When fixed overhead spending variance is unfavorable, it can be safely assumed that ________. fixed overhead allocated for actual output is lower than actual costs incurred fixed overhead allocated for actual output is higher than actual costs incurred flexible budget amount is higher than actual costs incurred flexible budget amount is lower than actual costs incurred

flexible budget amount is lower than actual costs incurred

Effective planning of variable overhead costs means that managers must identify the product advertising requirements and factor those into the variable overhead budget increase the expenditures in the variable overhead budgets increase the linearity between total costs and volume of production focus on activities that add value for the customer and eliminate nonvalue-added activities

focus on activities that add value for the customer and eliminate nonvalue-added activities

Work-in-process inventory would normally include

goods partially worked on but not yet fully completed

The cost function y = 300 + 2X ________. represents a fixed cost has an intercept of 300 is a nonlinear has a slope coefficient of 300

has an intercept of 300

An example of a denominator reason for calculating annual indirect-cost rates includes

higher levels of output demanded during the fall months

ABC systems create ________. activity-cost pools with a broad focus homogeneous activity-related cost pools activity-cost pools containing many direct costs one large cost pool

homogeneous activity-related cost pools

ABC systems create ________. activity-cost pools with a broad focus homogeneous activity-related cost pools one large cost pool activity-cost pools containing many direct costs

homogeneous activity-related cost pools

The learning-curve models presented in the text examine ________. how efficiency increases as more units are produced how quality increases over time the change in variable costs when quantity discounts are available how setup costs decline as more workers are added

how efficiency increases as more units are produced

Which of the following is another term for required rate of return? predetermined overhead rate total cost rate variance rate hurdle rate

hurdle rate

ABC systems help managers to ________. improve the inventory turnaround time value ending inventory more accurately identify new designs to reduce costs evaluate direct material costs more efficiently

identify new designs to reduce costs

Which of the following is the first stage to the capital budgeting process? obtain funding and make the investments selected forecast all potential cash flows attributable to the alternative projects identify potential capital investments that agree with the organization's strategy determine which investment yields the greatest benefit and the least cost to the organization

identify potential capital investments that agree with the organization's strategy

Which of the following is a stage of the capital budgeting process that indicates potential capital investments that agree with an organization's strategy

identify projects stage

Refining a cost system involves which of the following? creating as many cost pools as possible to capture all costs Seeking an easier and more cost effective way to calculate average costs classifying as many costs as indirect costs as is feasible identifying the activities involved in a process and understanding how those activities consume resources

identifying the activities involved in a process and understanding how those activities consume resources

One of the first steps to take when using CVP analysis to help make decisions is ________. estimating the volume of sales to make a good profit calculation of the degree of operating leverage for the company identifying the variable and fixed costs calculating the break-even point

identifying the variable and fixed costs

When comparing the operating incomes between absorption costing and variable costing, and ending finished inventory exceeds beginning finished inventory, it may be assumed that ________. variable cost per unit is more than fixed cost per unit absorption costing operating income exceeds variable costing operating income sales decreased during the period there is a favorable production-volume variance

absorption costing operating income exceeds variable costing operating income

A cost system determines the cost of a cost object by ________. accumulating costs accumulating and then assigning costs assigning and then accumulating costs assigning costs

accumulating and then assigning costs

The fundamental cost objects of ABC are ________. products cost drivers services activities

activities

The emphasis on variance analysis and its use in performance evaluation must be such that: Management should stretch resources to meet goals even if continuous improvement and quality suffers management should set targets that challenge but are reasonably achievable and require creativity and resourcefulness by personnel held accountable larger unfavorable variances should result in negative consequences for personnel held accountable managers focus on setting easy to attain targets

management should set targets that challenge but are reasonably achievable and require creativity and resourcefulness by personnel held accountable

To apply CVP analysis in a not-for profit organization ________. managers need to focus on the customer base rather than the cost drivers managers need to focus on measuring their output, which is different from the tangible units sold by manufacturing and merchandising companies managers need to focus on measuring their input, which is different from the tangible units consumed by manufacturing and merchandising companies managers need to focus on measuring their output, which is the same as tangible units sold by manufacturing and merchandising companies

managers need to focus on measuring their output, which is different from the tangible units sold by manufacturing and merchandising companies

Under GAAP, only ________ can be assigned to inventories in the financial statements. manufacturing costs historical costs period costs cost of goods sold

manufacturing costs

For a manufacturing-sector company, the cost of factory depreciation is classified as a ________. manufacturing overhead cost direct manufacturing labor cost direct material cost period cost

manufacturing overhead cost

Which of the following is an inventoriable cost? customer service costs marketing costs manufacturing overhead cost distribution costs

manufacturing overhead cost

In a job-costing system, a manufacturing firm typically uses an indirect-cost rate to estimate the ________ allocated to a job. total costs direct labor manufacturing overhead costs direct materials

manufacturing overhead costs

The variable overhead efficiency variance measures the difference between the ________, multiplied by the budgeted variable overhead cost per unit of the cost-allocation base. budgeted cost and the actual cost used to produce the actual output budgeted quantity of the cost-allocation base used and the budgeted quantity of the cost-allocation base that should have been used to produce the actual output actual cost incurred and the budgeted quantity of the cost-allocation base that should have been used to produce the actual output actual quantity of the cost-allocation base used and the budgeted quantity of the cost-allocation base that should have been used to produce the actual output

actual quantity of the cost-allocation base used and the budgeted quantity of the cost-allocation base that should have been used to produce the actual output

When variable overhead efficiency variance is favorable, it can be safely assumed that the ________. actual quantity of the cost-allocation base used is lower than the budgeted quantity actual quantity of the cost-allocation base used is higher than the budgeted quantity actual rate per unit of the cost-allocation base is higher than the budgeted rate actual rate per unit of the cost-allocation base is lower than the budgeted rate

actual quantity of the cost-allocation base used is lower than the budgeted quantity

Which of the following information is needed to prepare a flexible budget? actual variable cost actual units sold actual selling price per unit actual fixed cost

actual units sold

The ________ adjusts individual job-cost records to account for underallocated or overallocated overhead. adjusted allocation-rate weighted-average cost approach write-off to cost of goods sold approach proration approach

adjusted allocation-rate

Which of the following is a fixed cost for an automobile manufacturing plant? electricity used by assembly-line machines tires sales commissions administrative salaries

administrative salaries

For an automobile manufacturer, period costs include the cost of ________. assembly-line equipment the dashboard labor used for assembly advertising

advertising

Job-costing is likely to be used by________. Mortgage payment processors breakfast cereal producers oil refining companies advertising agencies

advertising agencies

Process costing ________. allocates all product costs, including materials, and labor is used exclusively in manufacturing results in different costs for different units produced is commonly used by general contractors who construct custom-built homes

allocates all product costs, including materials, and labor

Fixed costs depend on the ________. total number of units sold volume of production amount of unchanged costs for a given time period amount of resources used

amount of unchanged costs for a given time period

An efficiency variance reflects the difference between ________. an actual input quantity and a budgeted input quantity actual input quantities used last period and current period a standard input quantity in a company and its main competitors an actual input quantity used in a company and its main competitors

an actual input quantity and a budgeted input quantity

Which of the following companies will use a process costing system? an oil refining company an advertising firm a manufacturer of ships a custom kitchen cabinet company

an oil refining company

A plot of data that results in one extreme observation most likely indicates that ________. the cost-allocation base has been incorrectly identified an unusual event such as a plant shutdown occurred during that month more than one cost pool should be used individual cost items do not have the same cost driver

an unusual event such as a plant shutdown occurred during that month

Direct costs ________. are costs related to a particular cost object that cannot be traced to that cost object in a cost-effective manner are anything for which a measurement of costs is desired are costs related to a particular cost object that can be traced to that cost object in an economically feasible manner focus specifically on the costing needs of the CFO

are costs related to a particular cost object that can be traced to that cost object in an economically feasible manner

Under absorption costing, fixed manufacturing costs ________. are treated as an expense are period costs are inventoriable costs are sunk costs

are inventoriable costs

Nonfinancial performance measures ________. are often the sole basis of a manager's performance evaluations allow managers to make informed tradeoffs are rarely used to evaluate overall efficiency are usually used in combination with financial measures for control purposes

are usually used in combination with financial measures for control purposes

Management by exception is a practice whereby managers focus more closely on ________. areas not operating as anticipated and less closely on areas that are operating as anticipated unfavorable variances variances in the larger departments activity-based budgeting

areas not operating as anticipated and less closely on areas that are operating as anticipated

A cost function is a ________. process of calculating present value of projected cash flows mathematical description of how a cost changes with changes in the level of an activity relating to that cost is a very thorough and detailed way to identifying a cost object when there is a physical relationship between inputs and outputs process of allocating costs to cost centers or cost objects

mathematical description of how a cost changes with changes in the level of an activity relating to that cost

An unfavorable production-volume variance ________. indicates that the company had reduced its per unit fixed overhead cost to improve sales measures the amount of extra fixed costs planned for but not used is not a good measure of a lost production opportunity takes into account the effect of additional revenues due to maintaining higher prices

measures the amount of extra fixed costs planned for but not used

The only type of inventory that Macy's, a major U.S. department store chain would report on its balance sheet is: finished goods inventory direct materials inventory, work-in-process inventory, and finished goods inventory accounts merchandise inventory no inventory accounts as Macy's is a service-sector company

merchandise inventory

A manufacturing plant produces two product lines: golf equipment and soccer equipment. An example of a direct cost for the golf equipment line is ________. salaries of the clerical staff that work in the company administrative offices overheads incurred in producing both golf and soccer equipment beverages provided daily in the plant break room for the entire staff monthly lease payments for a specialized piece of equipment needed to manufacture the golf driver

monthly lease payments for a specialized piece of equipment needed to manufacture the golf driver

If the unit level of inventory increases during an accounting period, then ________. operating income will be the same under absorption costing and variable costing more operating income will be reported under absorption costing than variable costing less operating income will be reported under absorption costing than variable costing the exact effect on operating income cannot be determined

more operating income will be reported under absorption costing than variable costing

If a company is planning to reduce the selling price, they must believe that ________.

more units will be sold

In multiple regression, when two or more independent variables are highly correlated with one another, the situation is known as ________. heteroscedasticity autocorrelation multicollinearity homoscedasticity

multicollinearity

The difference between actual costing and normal costing is ________. actual costing uses actual quantities of cost-allocation bases normal costing uses actual quantities of direct-costs actual costing uses actual quantities of direct-costs normal costing uses budgeted indirect-costs

normal costing uses budgeted indirect-costs

The difference between actual costing and normal costing is ________. actual costing uses actual quantities of cost-allocation bases normal costing uses actual quantities of direct-costs actual costing uses actual quantities of direct-costs normal costing uses budgeted indirect-costs

normal costing uses budgeted indirect-costs

Cross-sectional data analysis includes ________. observing different entities during the same time period using the highest and lowest observation using a variety of time periods to measure the dependent variable comparing information in different cost pools

observing different entities during the same time period

Inventoriable costs are costs of a product that are considered ________. assets in a company's income statement when the costs are capitalized and that are expensed as cost of goods sold only when the product is sold assets in a company's balance sheet when the costs are incurred and that are expensed as cost of goods sold only when the product is sold liabilities in a company's balance sheet when the costs are incurred and that are expensed only when the product is sold liabilities in a company's income statement when the costs are capitalized and that are expensed only when the product is sold

assets in a company's balance sheet when the costs are incurred and that are expensed as cost of goods sold only when the product is sold

Cost assignment ________. associates accumulated costs with certain cost objects includes future and arbitrary costs is the difference between budgeted and actual costs is the same as cost accumulation

associates accumulated costs with certain cost objects

The payback method of capital budgeting approach to an investment decision

assumes that cash flows occur uniformly

The budgeted indirect-cost rate is calculated ________. at the end of each quarter during the year at the beginning of the year at the end of the year

at the beginning of the year

Most of the decisions determining the level of fixed overhead costs to be incurred will be made ________. by the end of a budget period at the start of a budget period by the middle of a budget period on a day-to-day ongoing basis

at the start of a budget period

A master budget is ________. developed at the end of a period based on the level of expected output at the start of the budget period a type of flexible budget once actual results are known a budget which starts from a zero base

based on the level of expected output at the start of the budget period

The process by which a company's products or services are measured relative to the best possible levels of performance is known as ________. variance analysis benchmarking a standard costing system efficiency

benchmarking

Allocating indirect costs to departments based on the relative revenue earned by those departments is done based on which of the following criterion? benefits received material resources used cause-and-effect relationships direct hours utilized

benefits received

The budgeted indirect-cost rate for each cost pool is computed as ________. budgeted annual indirect costs divided by budgeted actual quantity of cost allocation base budgeted annual indirect costs divided by budgeted annual quantity of cost allocation base budgeted annual quantity of cost allocation base divided by budgeted annual indirect costs actual annual indirect costs divided by budgeted annual quantity of cost allocation base

budgeted annual indirect costs divided by budgeted annual quantity of cost allocation base

The budgeted direct-labor cost rate includes ________. budgeted total direct-labor hours in the numerator budgeted total direct-labor costs in the denominator budgeted total direct-labor costs in the numerator budgeted total costs in indirect cost pool

budgeted total direct-labor costs in the numerator

The account analysis method estimates cost functions ________. using time-and-motion studies at a high cost, which renders it seldom used in a manner that cannot be usefully combined with any other cost estimation methods by classifying cost accounts as variable, fixed, or mixed based on qualitative analysis

by classifying cost accounts as variable, fixed, or mixed based on qualitative analysis

Alfred, owner of Hi-Tech Fiberglass Fabricators, Inc., is interested in using the reciprocal allocation method. The following data from operations were collected for analysis: Budgeted manufacturing overhead costs: Plant Maintenance PM (Support Dept) $380,000 Data Processing DP (Support Dept) $60,000 Machining M (Operating Dept) $210,000 Capping C (Operating Dept) $150,000 Services furnished: By Plant Maintenance (budgeted labor-hours): to Data Processing 3600 to Machining 5400 to Capping 8000By Data Processing (budgeted computer time): to Plant Maintenance 800 to Machining 4200 to Capping 600 What is the complete reciprocated cost of the Plant Maintenance Department? (Do not round any intermediary calculations.)

if I did this right 400693

A company would use multiple cost-allocation bases ________. if managers believe that using multiple cost-allocation bases is the only acceptable method because this is a simpler approach than using one cost allocation base because there is more than one way to allocate overhead if managers believed the benefits exceeded the additional costs of that costing system

if managers believed the benefits exceeded the additional costs of that costing system

Which of the following is a stage of the capital-budgeting process that tracks realized cash flows and compares those against estimated numbers? make predictions stage make decisions by choosing among alternatives stage identify projects stage implement the decision, evaluate performance, and learn stage

implement the decision, evaluate performance, and learn stage

Benchmarking is a process ________. in which the underlying processes of an organization is optimized using a systematic approach to achieve more efficient goals in which overhead costs are absorbed into units of output, or 'jobs' in which a firm's performance levels are compared against the best levels of performance in competing companies or in companies having similar processes which is based on calculating the breakeven point and analyzing the consequences of changes in various factors calculating the breakeven point

in which a firm's performance levels are compared against the best levels of performance in competing companies or in companies having similar processes

Period costs ________. should be treated as an indirect cost rather than as a direct manufacturing cost include the cost of selling, delivering, and after-sales support for customers include only fixed costs seldom influence financial success or failure

include the cost of selling, delivering, and after-sales support for customers

Problems with costing occur when ________. a computer screen requests an employee number before that employee is able to work on information related to a specific job bar coding is used to record materials used on the job incorrect job numbers are recorded on source documents incorrect product delivery forms are entered into the system

incorrect job numbers are recorded on source documents

The following information is for the Jeffries Corporation: Product A: Revenue $16.00 Variable Cost $12.00 Product B: Revenue $24.00 Variable Cost $16.00 Total fixed costs $75,000 Assume the sales mix consists of three units of Product A and one unit of Product B. If the sales mix shifts to four units of Product A and one unit of Product B, then the breakeven point will ________. increase cannot be determined from this information stay the same decrease

increase

Which of the following is the numerator in the mathematical expression for accrual accounting rate-of-return (AARR)

increase in expected average annual after-tax operating income

The following information is for Alex Corp: Product X: Revenue $15.00 Variable Cost $2.50 Product Y: Revenue $25.00 Variable Cost $10.00 Total fixed costs $50,000 If the sales mix shifts to one unit of Product X and two units of Product Y, then the weighted-average contribution margin will ________. stay the same decrease per unit decrease by $0.50 per unit increase per unit

increase per unit

The effect of spreading fixed manufacturing costs over a shrinking master-budget capacity utilization amount results in ________. increased unit costs greater demand for the product greater utilization of capacity more competitive selling prices

increased unit costs

Service companies, in particular, find great value from ABC because a vast majority of their cost structure is composed of ________ costs. factory indirect prime committed

indirect

The advantage of using normal costing instead of actual costing is ________. indirect costs are assigned at the end of the year when they are known the job cost is more accurate under normal costing indirect costs are assigned to a job on a timely basis normal costing provides a higher gross profit margin

indirect costs are assigned to a job on a timely basis

The advantage of using normal costing instead of actual costing is ________. the job cost is more accurate under normal costing normal costing provides a higher gross profit margin indirect costs are assigned to a job on a timely basis indirect costs are assigned at the end of the year when they are known

indirect costs are assigned to a job on a timely basis

Post-investment audits ________. are usually not feasible in a large project because the cost accounting system does not collect actual costs at the same level of detail as the initial plans had result in managers to overstate the expected cash inflows from projects and accept projects they should reject include obtaining appropriation requests so that the funding will be authorized to purchase the equipment provide management with feedback about the performance of a project

provide management with feedback about the performance of a project

The sales-volume variance is sometimes due to ________. unexpected increase in manufacturing labor time unexpected increase in the use of quantities of inputs of raw material quality problems leading to customer dissatisfaction the difference between selling price and budgeted selling price

quality problems leading to customer dissatisfaction

The slope of the line of regression is the ________. difference between the fixed cost and variable cost associated with the cost driver rate at which the dependent variable varies rate at which the independent variable varies difference between actual cost and estimated cost for each observation of the cost driver

rate at which the dependent variable varies

For a given job the direct costs associated with the job are ________. All sunk costs that can be traced to the job in an economically feasible way all fixed costs raw materials that can be traced to the job in an economically feasible way actual overhead that has been applied to the job

raw materials that can be traced to the job in an economically feasible way

Effective planning of variable overhead costs includes ________. choosing the appropriate level of investment reorganizing management structure eliminating value-added costs redesigning products or processes to use fewer resources

redesigning products or processes to use fewer resources

All else being equal, an increase in advertising expenditures will ________. increase variable costs reduce contribution margin increase selling price reduce operating income

reduce operating income

If cost accountants decide to use production of prior periods as a cost driver for repair cost of the current period they understand that there is no correlation between repair costs and levels of production repair costs tend to lag periods of high production cost accountants do not need to understand operations there is no cause-and-effect between production and repair costs

repair costs tend to lag periods of high production

The breakeven point is the activity level where ________. revenues equal the sum of variable and fixed costs revenues equal variable costs contribution margin equals total costs revenues equal fixed costs

revenues equal the sum of variable and fixed costs

Contribution margin equals ________. revenues minus product costs revenues minus variable costs revenues minus fixed costs revenues minus period costs

revenues minus variable costs

Which of the following costs will be treated as period costs under absorption costing? rent for factory building raw materials used in the production depreciation on factory equipment sales commission paid on sale of product

sales commission paid on sale of product

Gross margin is ________. contribution margin less variable costs sales revenue less cost of goods sold sales revenue less variable costs contribution margin less fixed costs

sales revenue less cost of goods sold

The contribution margin per unit equals . fixed cost - contribution margin ratio selling price - variable costs per unit selling price - costs of good sold selling price - fixed costs per unit

selling price - variable costs per unit

The following conditions with departmental indirect-cost rates will result in allocation of costs similar to ABC costing except: a single activity accounts for a sizable portion of the costs of the department several activities cause a sizable portion of the costs of the department Significant costs are incurred on different activities within a department significant costs are incurred for different activities with different cost -allocation bases within a department

several activities cause a sizable portion of the costs of the department

Which of the following are reasons for using longer periods, such as a year, to calculate indirect cost rates? longer the period, the greater is the influence of seasonal patterns on the amount of costs shorter the period, the greater is the influence of seasonal patterns on the amount of costs shorter the period, the smaller is the influence of seasonal patterns on the amount of opportunity costs longer the period, the smaller is the influence of seasonal patterns on the amount of opportunity costs

shorter the period, the greater is the influence of seasonal patterns on the amount of costs

Managers and accountants collect most of the cost information that goes into their systems through ________. computer programs an information data bank source documents time surveys

source documents

Ways to "produce for inventory" that result in increasing operating income include ________. switching production to products that absorb the most amounts of fixed manufacturing costs switching production to products that absorb the least amounts of fixed manufacturing costs undervaluing ending inventory by not recording certain costs that have been incurred delaying items that absorb the greatest amount of fixed manufacturing costs

switching production to products that absorb the most amounts of fixed manufacturing costs

A learning curve is a function ________. where unit costs increase as productivity increases that increases at a greater rate as workers become more familiar with their tasks that is linear that measures the decline in labor-hours per unit due to workers becoming better at a job

that measures the decline in labor-hours per unit due to workers becoming better at a job

A cost driver should be measurable and have an economically plausible relationship with the dependent variable which means: that the cost driver can be identified in an economically feasible way that the relationship is based on correlation that the relationship must be based on a physical relationship that the relationship is based on a cause -and-effect criterion and makes economic sense to management

that the relationship is based on a cause -and-effect criterion and makes economic sense to management

Which cost estimation method would involve analyzing direct labor subsidiary accounts and classifying costs as as variable, fixed, or mixed to derive cost estimation formulas? the conference method the incremental costing method the account analysis method the marginal costing method

the account analysis method

An unfavorable variance indicates that ________. the actual revenues exceed the budgeted revenues the actual units sold are less than the budgeted units the actual costs are less than the budgeted costs the budgeted contribution margin is more than the actual amount

the actual units sold are less than the budgeted units

A $5,000 unfavorable flexible-budget variance indicates that ________. the flexible-budget amount exceeded actual variable manufacturing overhead by $5,000 the flexible-budget amount exceeded standard variable manufacturing overhead by $5,000 the actual variable manufacturing overhead exceeded the flexible-budget amount by $5,000 the standard variable manufacturing overhead exceeded the flexible-budget amount by $5,000

the actual variable manufacturing overhead exceeded the flexible-budget amount by $5,000

Cost allocation is ________. made based on material acquisition document the process of determining the opportunity cost of a cost object chosen the process of tracking both direct and indirect costs associated with a cost object the assignment of indirect costs to the chosen cost object

the assignment of indirect costs to the chosen cost object

With a step fixed-cost function ________. fixed cost changes proportionally with the level of activity fixed cost is often approximated with a continuous variable-cost function the cost remains the same over wide ranges of the activity in each relevant range the cost varies with the changes in the activity

the cost remains the same over wide ranges of the activity in each relevant range

Which of the following is a learning-curve model? the account analysis learning model and the conference learning method model the simple regression model and the multiple regression model the cumulative average-time learning model and the incremental unit-time learning model the multicollinearity learning model and the goodness of fit learning model

the cumulative average-time learning model and the incremental unit-time learning model

Which of the following is a learning-curve model? the multicollinearity learning model and the goodness of fit learning model the cumulative average-time learning model and the incremental unit-time learning model the account analysis learning model and the conference learning method model the simple regression model and the multiple regression model

the cumulative average-time learning model and the incremental unit-time learning model

Effectiveness is ________. the continuous process of comparing a firm's performance levels against the best levels of performance in competing companies is a practice whereby managers focus more closely on areas that are not operating as expected and less closely on areas that are the relative amount of inputs used to achieve a given output level the degree to which a predetermined objective or target is met

the degree to which a predetermined objective or target is met

A variance is ________. the difference between actual fixed cost per unit and standard variable cost per unit the difference between actual variable cost per unit and standard fixed cost per unit the standard units of inputs for one output the difference between an actual result and a budgeted performance

the difference between an actual result and a budgeted performance

Which cost estimation method uses time-and-motion studies to reveal that to make a high quality men's suit jacket , it takes of 3 hours of direct labor effort per jacket and 5 minutes of a salaried manager to perform quality control? the high-low method the cash accounting method the industrial engineering method the accrual accounting method

the industrial engineering method

An example of a sunk cost in a capital budgeting decision for new equipment is

the original price of an old equipment

An unfavorable fixed overhead spending variance indicates that ________. the denominator level was more than planned the fixed overhead cost-allocation base was not used efficiently there was more excess capacity than planned the price of fixed overhead items cost more than budgeted

the price of fixed overhead items cost more than budgeted

An unfavorable fixed overhead spending variance indicates that ________. the fixed overhead cost-allocation base was not used efficiently there was more excess capacity than planned the denominator level was more than planned the price of fixed overhead items cost more than budgeted

the price of fixed overhead items cost more than budgeted

The spreading of underallocated or overallocated overhead among ending work-in-process, finished goods, and cost of goods sold is called ________. the adjusted allocation rate approach the weighted-average cost approach the proration approach the write-off of cost of goods sold approach

the proration approach

Which of the following approaches spreads underallocated or overallocated overhead among ending balances in Work-in-Process Control, Finished Goods Control, and Cost of Goods Sold? the write-off variances to cost of goods sold approach the adjusted allocation-rate approach the reinstatement approach the proration approach

the proration approach

Which of the following could be a reason for a favorable material price variance? the purchasing manager giving orders for small quantity to reduce storage cost the purchasing manager accepting a bid from the highest-priced supplier to ensure the quality of material the purchasing manager bargaining effectively with suppliers the personnel manager hiring underskilled workers

the purchasing manager bargaining effectively with suppliers

Which cost estimation method uses a formal statistical method such as regression analysis to develop cost functions based on past data? the conference method the accrual accounting method the quantitative analysis method the cash accounting method

the quantitative analysis method

Efficiency is ________. the difference between an actual input quantity and a budgeted input quantity the relative amount of inputs used to achieve a given output level the degree to which a predetermined objective or target is met the continuous process of comparing a firm's performance levels against the best levels of performance in competing companies

the relative amount of inputs used to achieve a given output level

The variable overhead efficiency variance is computed ________ and interpreted ________ the direct-cost efficiency variance

the same as; differently than

The breakeven point decreases if ________. the selling price per unit decreases the contribution margin per unit decreases the total fixed costs decrease the variable cost per unit increases

the total fixed costs decrease

The budgeted fixed manufacturing cost rate is the lowest for ________. theoretical capacity master-budget capacity utilization practical capacity normal capacity utilization

theoretical capacity

Which of the following might explain why one manager might assign certain costs to a cost object while a different manager might assign some different costs to that same cost object? one manager might classify one cost as direct while another might classify that same cost as indirect one manager might be pricing a cost object while the other manager might be seeking the inventoriable cost one manager might value an inventory item for the balance sheet as the sum of all the value chain costs while another manager might choose to only recognize variable costs as inventoribale costs for GAAP purposes. For financial statement purposes, one manager might include all the costs of the value chain while the other might only only include GAAP cost

one manager might be pricing a cost object while the other manager might be seeking the inventoriable cost

For a manufacturing company, direct labor costs would be included in ________. only in the merchandise inventory to reflect the value added in the production process only the direct materials inventory to reflect the cost of labor that has worked using those materials only in both work-in-process inventory and finished goods inventory direct materials inventory, work-in-process inventory, and finished goods inventory accounts

only in both work-in-process inventory and finished goods inventory

Customers expect to pay a price that includes ________. only the cost of actual capacity used the cost of unused capacity the cost of direct materials, direct labor, and fixed overhead variable costs but not capacity costs

only the cost of actual capacity used

In the merchandising sector ________. fixed overhead costs are subtracted to determine contribution margin only variable costs are subtracted to determine gross margin fixed overhead costs are subtracted to determine gross margin all operating costs are subtracted to determine contribution margin

only variable costs are subtracted to determine gross margin

Assume only the specified parameters change in a cost-volume-profit analysis. If the contribution margin increases by $6 per unit, then ________. operating income increases by $6 per unit fixed costs decreases by $6 per unit operating income decreases by $6 per unit fixed costs increases by $6 per unit

operating income increases by $6 per unit

At breakeven point, ________. operating income is equal to zero breakeven revenues equal fixed costs divided by the variable cost per unit contribution margin minus fixed costs is equal to profits earned revenues equal fixed costs minus variable costs

operating income is equal to zero

In multiproduct situations, when sales mix shifts toward the product with the lowest contribution margin then ________. total contribution margin will increase total revenues will increase interest cost will decrease operating income will decrease

operating income will decrease

In multiproduct situations, when sales mix shifts toward the product with the lowest contribution margin then ________. total contribution margin will increase total revenues will increase operating income will decrease interest cost will decrease

operating income will decrease

Using activity-cost rates rather than department indirect-cost rates to allocate costs results in different product costs when ________. there are several homogeneous cost pools different activities have the same cost-allocation base a single activity accounts for a sizable portion of department costs different products use different resources in the same proportion

there are several homogeneous cost pools

A single indirect-cost rate distorts product costs because ________. competitive pricing is ignored it assumes all costs are product costs it recognizes specific activities that are required to produce a product there is an assumption that all support activities affect all products in a uniform way

there is an assumption that all support activities affect all products in a uniform way

If unit outputs exceed the breakeven point ________. total sales revenue will exceed variable costs total sales revenue will exceed fixed costs there will be a profit there will be an increase in fixed costs

there will be a profit

The reason to have a post-investment audit is ________. they encourage mid-level managers to make overly optimistic estimates during the early stages of the capital budgeting process they help in calculating present value they analyze by calculating contribution-margin they help alert senior management to problems in the implementation of projects

they help alert senior management to problems in the implementation of projects

When "available time" (i.e., setup-hours) is used to calculate a cost of a resource and to allocate costs to cost objects , the system is called: job costing process costing hybrid costing time-driven activity based costing

time-driven activity based costing

One reason for assigning selling and distribution costs to products for analytical purposes is ________. to justify a varied product-mix that these costs should be included in the cost of goods sold in the income statement that all direct costs must be assigned to ensure that all costs are considered

to ensure that all costs are considered

Many companies have switched from absorption costing to variable costing for internal reporting ________. so the denominator level is more accurate to comply with external reporting requirements as required by GAAP to reduce the undesirable incentive to build up inventories that would show higher operating income to increase bonuses for managers

to reduce the undesirable incentive to build up inventories that would show higher operating income

Which of the following is true if the volume of sales increases (within a relevant range)? total variable cost decreases total variable cost increases total fixed cost increases total fixed cost decreases

total variable cost increases

A cost is considered direct if it can be traced to a particular cost object in a cost effective way which means it can be traced in a manner that is accurate possibly traced accurately with an investment in hardware and software traced in an economically feasible way traced easily with the aid of technology

traced in an economically feasible way

With traditional costing systems, products manufactured in small batches and in small annual volumes may be ________ because batch-related and product-sustaining costs are assigned using unit-related drivers. fairly costed undercosted overcosted ignored

undercosted

A favorable price variance for direct manufacturing labor might indicate that ________. unexpected increase in direct labor rates congestion due to scheduling problems employees were paid more than planned underskilled employees are being hired

underskilled employees are being hired

Process costing is ________. used by businesses when manufacturing goods above normal capacity used by businesses to price identical products used to enhance employees' job satisfaction used by businesses to price unique products or identical products produced in batches

used by businesses to price identical products

Job costing is ________. used by businesses to price unique products for different jobs used to calculate the percentage of work completed used to calculate equivalent units used by businesses to price identical products

used by businesses to price unique products for different jobs

While calculating the costs of products and services, a standard costing system ________. uses standard costs to determine the cost of products traces direct costs to output by multiplying the standard prices or rates by the actual quantities does not keep track of overhead cost allocates overhead costs on the basis of the actual overhead-cost rates

uses standard costs to determine the cost of products

Quantitative analysis methods estimate cost functions ________. using the pooling of knowledge from each value chain function based on analysis and opinions gathered from various departments using the time-and-motion studies using a formal mathematical method to fit cost functions to past data observations

using a formal mathematical method to fit cost functions to past data observations

An unfavorable flexible-budget variance for variable costs may be the result of ________. using more input quantities than were budgeted paying lower prices for inputs than were budgeted selling output at a higher selling price than budgeted selling less quantity compared to the budgeted

using more input quantities than were budgeted

Which of the following costs is inventoried when using absorption costing? fixed selling costs fixed administrative costs variable selling costs variable manufacturing costs

variable manufacturing costs

When a greater proportion of costs are fixed costs, then ________. a decrease in sales reduces the cost per unit a decrease in sales reduces the total fixed cost per unit a small increase in sales results in a small decrease in operating income when demand is low the risk of loss is high

when demand is low the risk of loss is high

When a greater proportion of costs are fixed costs, then ________. a decrease in sales reduces the total fixed cost per unit a decrease in sales reduces the cost per unit a small increase in sales results in a small decrease in operating income when demand is low the risk of loss is high

when demand is low the risk of loss is high

Classifying a cost as either direct or indirect depends upon ________. whether the cost is expensed in the period in which it is incurred the behavior of the cost in response to volume changes whether a cost is fixed or variable whether the cost can be traced to a particular cost object in an economically feasible way

whether the cost can be traced to a particular cost object in an economically feasible way

The approach often used when dealing with small amounts of underallocated or overallocated overhead is the ________. write-off to cost of goods sold approach adjusted write-off approach proration approach adjusted allocation-rate approach

write-off to cost of goods sold approach

Which of the following is an equation of a fixed cost function? y = a + bX y = (a + b)X y = bX y = a

y = a

Bennet Company employs 20 individuals. Eighteen employees are paid $18 per hour and the rest are salaried employees paid $3,000 a month. Which of the following is the total cost function of personnel? y = a + bX y = a y = b y = bX

y = a + bX

Tony Manufacturing produces a single product that sells for $80. Variable costs per unit equal $50. The company expects total fixed costs to be $82,000 for the next month at the projected sales level of 2,800 units. In an attempt to improve performance, management is considering a number of alternative actions. Each situation is to be evaluated separately. Suppose that management believes that a 14% reduction in the selling price will result in a 14% increase in sales. If this proposed reduction in selling price is implemented

operating income will decrease by $23,990

Which of the following influences the make or buy decision to the company? sunk cost historical cost plant depreciation opportunity cost

opportunity cost

the term for understanding why actual performance deviates from planned performance is. favorable variance organizational learning continuous improvement variance calculation

organizational learning

Many government agency contracts exclude all but which of the following costs from reimbursement under cost-plus agreements? part of the research and development costs production costs design costs marketing costs

part of the research and development costs

Which of the following is an example of nonfinancial performance measure? quantity discounts obtained on order of large quantity direct manufacturing labor efficiency variance percentage of products started and completed without requiring any rework direct materials price variance

percentage of products started and completed without requiring any rework

The income statement of a service-sector firm reports ________. direct labor costs cost of goods sold period costs only both period costs and cost of of goods sold

period costs only

Which of the following represents cross-sectional data? number of machine-hours used for the past 10 years personnel costs of a month at 10 different organizations indirect manufacturing labor costs for the past 5 years maintenance cost of machine in a plant for the past 3 years

personnel costs of a month at 10 different organizations

The flexible-budget variance for direct cost inputs can be further subdivided into a ________. static-budget variance and a sales-volume variance sales-volume variance and an efficiency variance price variance and an efficiency variance static-budget variance and a price variance

price variance and an efficiency variance

Activity-based costing information can be used for ________. product-mix decisions advertisement decisions pricing decisions inventory valuation

pricing decisions

Overcosting a particular product may result in: pricing the product too low pricing the product too high operating efficiencies understating total product costs

pricing the product too high

If indirect-cost rates are calculated monthly, distortions might occur because of ________. routine monthly preventive-maintenance costs that benefit future months property tax payments made in July and December salary hikes at the beginning of the financial year rental costs paid monthly

property tax payments made in July and December

When benchmarking, management accountants are most valuable when they ________. provide insight into why costs or revenues differ across companies present differences in the benchmarking data to management provide complex mathematical analysis highlight differences in the benchmarking data to management

provide insight into why costs or revenues differ across companies

________ method includes fixed manufacturing overhead costs as inventoriable costs. Throughput costing Activity-based costing Variable costing Absorption costing

Absorption costing

Which of the following statements is true of ABC systems? ABC system employs multiple activity-cost drivers. ABC system will always result in higher product costs. ABC system is least suited for service companies. ABC system is simpler compared to traditional systems.

ABC system employs multiple activity-cost drivers.

Aqua Company produces two products-Alpha and Beta. Alpha has a high market share and is produced in bulk. Production of Beta is based on customer orders and is custom designed. Also, 55% of Beta's cost is shared between design and setup costs, while Alpha's major portions of costs are direct costs. Alpha is using a single cost pool to allocate indirect costs. Which of the following statements is true of Aqua? Aqua will undercost Alpha's indirect costs because alpha has high direct costs. Aqua will overcost Beta's direct costs as it is using a single cost pool to allocate indirect costs. Aqua will overcost Alpha's indirect costs as it is using a single cost pool to allocate indirect costs. Aqua will overcost Beta's indirect costs because beta has high indirect costs.

Aqua will overcost Alpha's indirect costs as it is using a single cost pool to allocate indirect costs.

________ is considered while choosing a cost allocation base for activity costs in ABC costing. Market share of a product Availability of reliable data and measures Marketing strategy and material price level Product price level

Availability of reliable data and measures

Which of the following is the mathematical expression for the budgeted fixed overhead cost per unit of cost allocation base? Budgeted fixed overhead cost per unit of cost allocation base = Actual total costs in fixed overhead cost pool ÷ Actual total quantity of cost allocation base Budgeted fixed overhead cost per unit of cost allocation base = Budgeted total costs in fixed overhead cost pool ÷ Actual total quantity of cost allocation base Budgeted fixed overhead cost per unit of cost allocation base = Actual total costs in fixed overhead cost pool ÷ Budgeted total quantity of cost allocation base Budgeted fixed overhead cost per unit of cost allocation base = Budgeted total costs in fixed overhead cost pool ÷ Budgeted total quantity of cost allocation base

Budgeted fixed overhead cost per unit of cost allocation base = Budgeted total costs in fixed overhead cost pool ÷ Budgeted total quantity of cost allocation base

Which of the following mathematical expression is used to calculate budgeted variable overhead cost rate per output unit? Budgeted input allowed per output unit ÷ Budgeted variable overhead cost rate per input unit Budgeted output allowed per input unit × Budgeted variable overhead cost rate per input unit Budgeted input allowed per output unit × Budgeted variable overhead cost rate per input unit Budgeted output allowed per input unit ÷ Budgeted variable overhead cost rate per input unit

Budgeted input allowed per output unit × Budgeted variable overhead cost rate per input unit

Which of the following companies is part of the merchandising sector of our economy? KPMG Whole Foods Market Jaguar Hewlett Packard

Whole Foods Market

Variable costing regards fixed manufacturing overhead as a(n) ________. product cost administrative cost inventoriable cost period cost

period cost

Under GAAP, for the purposes of calculating inventory costs, product costs include ________. design costs all costs incurred along the value chain only inventoriable costs only research and development costs

only inventoriable costs

The Speedjet Aircraft Corporation has a central materials laboratory. The laboratory has only two users, the Large Plane Department and the Small Plane Department. The following data apply to the coming budget year: Budgeted costs of operating the materials laboratory for 150,000 to 250,000 technician hours per year: Fixed costs per year $8,000,000 Variable costs $70​ per technician hourBudgeted long-run usage in hours per year: Large Plane Department 90,000​ technician hours Small Plane Department 70,000​ technician hours Budgeted amounts are used to calculate the allocation rates. Actual usage for the year by the Large Plane Department was 80,000 technician hours and by the Small Plane Department was 120,000 technician hours. If a dual-rate cost-allocation method is used, what amount of materials laboratory costs will be allocated to the Large Plane Department? Assume budgeted usage is used to allocate fixed materials laboratory costs and actual usage is used to allocate variable materials laboratory costs.

10100000 Allocation of materials laboratory costs to Large plane department: Fixed cost: 90,000/160,000×$8,000,000=$4,500,000 Variable cost:80,000 tech. Hours ×$70 per tech. Hours=$5,600,000 Total cost allocated to Large plane=$10,100,000

The Speedjet Aircraft Corporation has a central materials laboratory. The laboratory has only two users, the Large Plane Department and the Small Plane Department. The following data apply to the coming budget year: Budgeted costs of operating the materials laboratory for 100,000 to 200,000 technician hours per year: Fixed costs per year $8,200,000 Variable costs $74​ per technician hourBudgeted long-run usage in hours per year: Large Plane Department 90,000​ technician hours Small Plane Department 110,000​ technician hours Budgeted amounts are used to calculate the allocation rates. Actual usage for the year by the Large Plane Department was 70,000 technician hours and by the Small Plane Department was 80,000 technician hours. If a dual-rate cost-allocation method is used, what amount of materials laboratory costs will be budgeted for the Small Plane Department?

12650000 110/200*8200000 + 110000*74

Ambinu Flower Company provides flowers and other nursery products for decorative purposes in medium to large sized restaurants and businesses. The company has been investigating the purchase of a new specially equipped van for deliveries. The van has a value of $133,750 with a six-year life. The expected additional cash inflows are $52,500 per year. What is the payback period for this investment?

133750/52500=2.5

A recent college graduate has the choice of buying a new car for $33,500 or investing the money for four years with an 11% expected annual rate of return. He has an investment of $41,000 in equities and bonds which yields 8% expected annual rate of return. If the graduate decides to purchase the car, the best estimate of the opportunity cost of that decision is ________. $3,280 $14,740 $41,000 $18,040

14,740

Planet Design Services, Inc., is considering replacing a machine. The following data are available: Replacement Old Machine Machine Original cost $650,000​ $510,000 Useful life in years 10​ 5 Current age in years 5​ 0 Book value $400,000​ — Disposal value now $142,000​ — Disposal value in 5 years 0 0 Annual cash operating costs $100,000​ $66,000 The difference between keeping the old machine and replacing the old machine is ________.

198000 in favor of KEEPING the old machine New 510000+(5*66000)=840000 Old 142000+(5*100000)=642000 (faster and better pay off with old)

Which of the following methods utilizes discounted cash flows when analyzing potential capital expenditures? Methods: 1. Accrual accounting rate-of-return 2. Internal Rate of Return (IRR) 3. Payback Period 4. Net Present Value (NPV)

2 and 4

Excellent Manufacturers Inc. has a current production level of 20,000 units per month. Unit costs at this level are: Direct materials $0.26 Direct labor 0.40 Variable overhead 0.16 Fixed overhead 0.21 Marketing: fixed 0.25 Marketing/distribution: variable 0.42 Current monthly sales are 18,000 units. Jax Company has contacted Excellent about purchasing 1,550 units at $2.00 each. Current sales would NOT be affected by the one-time-only special order, and variable marketing/distribution costs would NOT be incurred on the special order. What is Ratzlaff Company's change in operating profits if the special order is accepted?

2-.26-.4-.16=1.18*1550=1829 increase

Forise Water Company drills small commercial water wells. The company is in the process of analyzing the purchase of a new drill. Information on the proposal is provided below. Initial investment: Asset $700,000 Working capital $188,000 Operations (per year for four years): Cash receipts $530,000 Cash expenditures $240,000 Disinvestment: Salvage value of drill (existing) $120,000 Discount rate 18% What is the net present value of the investment? Assume there is no recovery of working capital.

200100-188000=12100

Pearl Manufacturing Company provides glassware machines for major department store retailers. The company has been investigating a new piece of machinery for its production department. The old equipment has a remaining life of six years and the new equipment has a value of $319,400 with a six-year life. The expected additional cash inflows are $113,000 per year. What is the payback period for this investment?

319400/113000=2.8

Johnson Superior Products Inc. produces hospital equipment and the setup requirements vary from product to product. Johnson produces its products based on customer orders and uses ABC costing. In one of its indirect cost pools, setup costs and distribution costs are pooled together. Costs in this pool are allocated using number of customer orders for the easiness of costing operations. Based on the information provided, which of the following arguments is valid? Johnson has failed to use the correct cost driver as the cost-allocation base for setup costs. Johnson has clearly failed to identify as many direct costs as is economically feasible. Johnson has unnecessarily wasted resources by classifying setup and distribution costs as they could have been considered as direct costs. All costs in a homogeneous cost pool have the same or a similar cause-and-effect relationship with the single cost driver that is used as the cost-allocation base for Johnson.

Johnson has failed to use the correct cost driver as the cost-allocation base for setup costs.

Which of the following companies is part of the service sector of our economy? Macy's Amazon.com KPMG Apple Inc.

KPMG

Fixed overhead costs include ________. indirect materials energy costs Leasing of machinery used in a factory the cost of sales commissions

Leasing of machinery used in a factory

________ is an example of an output unit-level cost in the cost hierarchy. Machine depreciation Factory rent expense Top management compensation costs Building security costs

Machine depreciation

Which of the following is true of variance? Managers should interpret a favorable variance as "good news". Managers should not simply interpret a favorable variance as good but should understand why the variance occurred. A small variance or zero variance definitively reveals efficient performance. Managers' performance must be evaluated solely on single variance.

Managers should not simply interpret a favorable variance as good but should understand why the variance occurred.

________ sector companies purchase materials and components and convert them into finished goods. Manufacturing Service Merchandising Professional

Manufacturing

Actual (rather than allocated) manufacturing overhead costs are first recorded in the ________. Manufacturing Overhead Control account Work-in-Process Control account Finished Goods Control account Cost of Goods Sold account

Manufacturing Overhead Control account

Which account is credited if direct materials of $28,000 and indirect materials of $7,000 are sent to the manufacturing plant floor? Accounts Payable Control for $21,000 Manufacturing Overhead Control for $35,000 Materials Control for $35,000 Work-in-Process Control for $35,000

Materials Control for $35,000

Springer Products manufactures three different product lines, Model X, Model Y, and Model Z. Considerable market demand exists for all models. The following per unit data apply: Model X Model Y Model Z Selling price $55​ $69​ $78 Direct materials 10​ 10​ 10 Direct labor ($15 per hour) 15​ 15​ 30 Variable support costs ($7 per machine-hour) 7​ 14​ 14 Fixed support costs 11​ 11​ 11 If there is a machine breakdown, which model is the most profitable to produce? Model X Model Y Model Z Both Model X and Model Y have same and highest profitability

Model X

If management experiences an unfavorable direct materials efficiency variance, which of the following would not be the possible corrective action? Negotiate lower prices for material acquisition Purchase higher quality materials Improve the design of the product Provide additional training for the direct laborers

Negotiate lower prices for material acquisition

Which of the following companies is part of the manufacturing sector of our economy? Nike Corvette Law Firm Barnes & Noble Sears, Roebuck, and Company

Nike

________ is based on the level of capacity utilization that satisfies average customer demand over periods generally longer than one year. Theoretical capacity Practical capacity Normal capacity utilization Master-budget capacity utilization

Normal capacity utilization

A company has three products possible products that it can produce in a machine intensive production process. Capacity is constrained by the number of hours the machines can run during a period and the products are so popular that all units produced will be sold. Here is additional information: CM Per Unit A: 20 B:30 C:40 Machine Hours Per Unit A:2.5 B:3.25 C:4.5 Which of the following would be an accurate conclusion based on these facts Since A takes less time to produce, maximization of operating income will occur by emphasizing production and sales of A. Product B should be emphasized if the goal is to maximize contribution margin. A balanced mix of 1/3 A, 1/3 B, and 1/3 C should be the goal when maximizing operating income in the short-run. Since Product C has the greatest contribution margin per unit and therefore emphasizing its production and sales will lead to the highest operating income in the short-run

Product B should be emphasized if the goal is to maximize contribution margin CM per hour: A:8 B:9.23 C:8.9

________ are the costs of activities undertaken to support individual products or services regardless of the number of units or batches in which the units are produced. Facility-sustaining costs Product-sustaining costs Unit-level costs Batch-level costs

Product-sustaining costs

Springer Products manufactures three different product lines, Model X, Model Y, and Model Z. Considerable market demand exists for all models. The following per unit data apply: Model X Model Y Model Z Selling price $56​ $68​ $78 Direct materials 7​ 7​ 7 Direct labor ($16 per hour) 16​ 16​ 32 Variable support costs ($7 per machine-hour) 7 15​ 14 Fixed support costs 15​ 15​ 15 How can Lisa Dynondo encourage her salespeople to promote the more profitable model? Provide higher sales commissions for items which has the lowest cost and lower sales commissions for items with highest cost. Provide higher sales commissions for higher priced items. Put all sales persons on fixed salary. Provide higher sales commissions for items with the greatest contribution margin per constrained resource.

Provide higher sales commissions for items with the greatest contribution margin per constrained resource.

Which of the following is not an inventoriable cost of a manufacturer? Sales commissions paid to sales representatives who sell the products made by the manufacturer Wages paid to assembly-line workers Plant depreciation incurred Property taxes on plant

Sales commissions paid to sales representatives who sell the products made by the manufacturer

Which of the following statements shows a difference between simple regression and multiple regression? Simple regression uses only one dependent variable and more than one independent variables, whereas multiple regression uses more than one dependent variable and only one independent variable. Simple regression uses more than one dependent and independent variables, whereas multiple regression uses only one dependent and independent variable. Simple regression uses only one dependent and one independent variable, whereas multiple regression uses one dependent and more than one independent variable. Simple regression uses only the independent variables, whereas multiple regression uses only dependent variables.

Simple regression uses only one dependent and one independent variable, whereas multiple regression uses one dependent and more than one independent variable.

Which of the following would be considered an actual cost of a current period? The $22 of direct material cost per unit assumed in the actual budget of a manufacturer of chairs The $25 of materials in a manufactured chair that is ready to be shipped to the customer The average of historical material cost data for a chair manufactured in several past accounting periods The expected cost of materials for a chair as a result of engineering specifications

The $25 of materials in a manufactured chair that is ready to be shipped to the customer

Which of the following would be considered an actual cost of a current period? The average of historical material cost data for a chair manufactured in several past accounting periods The expected cost of materials for a chair as a result of engineering specifications The $22 of direct material cost per unit assumed in the actual budget of a manufacturer of chairs The $25 of materials in a manufactured chair that is ready to be shipped to the customer

The $25 of materials in a manufactured chair that is ready to be shipped to the customer

Which of the following statements is true of accrual accounting rate of return (AARR) method and internal rate of return (IRR) method? The AARR method calculates the return considering the time value of money, whereas the IRR method calculates the return ignoring the time value of money. The AARR method considers cash flows, whereas the IRR method considers operating income. The AARR method calculates the return using operating-income numbers after considering accruals and taxes, whereas the IRR method calculates the return using after-tax cash flows and the time value of money. AARR method calculates the return in absolute terms, whereas IRR method calculates the result in terms of percentage.

The AARR method calculates the return using operating-income numbers after considering accruals and taxes, whereas the IRR method calculates the return using after-tax cash flows and the time value of money.

Which of the following is a step to overcome problems related to data collection for estimating cost function? The analyst should also use extreme values while calculating cost functions. The analyst should consider fixed costs as variable. The analyst should remove the inflationary effects. The analyst should not use accrual accounting.

The analyst should remove the inflationary effects.

Which of the following involves the process of making decisions for significant financial investments in projects to develop new products, expand production capacity, or remodel current production facilities?

capital budgeting

Upon which of the following items does discounted cash flow methods for capital budgeting focus

cash inflows and required rate of return

Net initial investment includes ________. cash outflow to purchase new equipment, depreciation on new equipment, and after-tax cash inflow from disposal of the old equipment cash outflow to purchase new equipment, cash outflow for working capital, and depreciation on new equipment depreciation on new equipment, cash outflow for working capital, and after-tax cash inflow from disposal of the old equipment cash outflow to purchase new equipment, cash outflow for working capital, and after-tax cash inflow from disposal of the old equipment

cash outflow to purchase new equipment, cash outflow for working capital, and after-tax cash inflow from disposal of the old equipment

The major challenge when planning fixed overhead is ________. calculating total costs calculating the cost-allocation rate choosing the appropriate planning period choosing the appropriate level of capacity

choosing the appropriate level of capacity

Compared to variable overhead costs planning, fixed overhead cost planning has an additional strategic issue beyond undertaking only essential activities and efficient operations. That additional requirement is best described as: identifying essential value-adding activities choosing the appropriate level of capacity that will benefit the company in the long-run focusing on the highest possible quality increasing the linearity between total costs and volume of production

choosing the appropriate level of capacity that will benefit the company in the long-run

Effective planning of fixed overhead costs includes ________. eliminating value-added costs choosing the appropriate level of investment in productive assets planning day-to-day operational decisions determining which products are to be produced

choosing the appropriate level of investment in productive assets

Effective planning of fixed overhead costs includes ________. determining which products are to be produced choosing the appropriate level of investment in productive assets eliminating value-added costs planning day-to-day operational decisions

choosing the appropriate level of investment in productive assets

If each motorcycle requires a belt that costs $20 and 2,000 motorcycles are produced for the month, the total cost for belts is ________. considered to be an indirect fixed cost considered to be a direct fixed cost considered to be a direct variable cost considered to be an indirect variable cost

considered to be a direct variable cost

The selling price per unit less the variable cost per unit is the ________. contribution margin per unit margin of safety fixed cost per unit gross margin

contribution margin per unit

Which of the following includes both traced direct costs and allocated indirect costs? cost pools cost tracing cost allocations cost assignments

cost assignments

The determination of a cost as either direct or indirect depends upon the ________. cost object chosen tax system chosen inventory valuation accounting standards

cost object chosen

A ________ is a grouping of individual indirect cost items. cost-allocation base cost assignment job-costing system cost pool

cost pool

Assigning direct costs to a cost object is called ________. cost assignment cost allocation cost tracing cost pooling

cost tracing


Conjuntos de estudio relacionados

Anatomie Biomedische wetenschappen KuLeuven 2021

View Set

HLWL 2230 Stress Management Quiz #1

View Set

Interpersonal Communication: Everyday Encounters (Chapter 1)

View Set

Chapter 12 - Supply Chain Management in the Service Industry

View Set

Adolescent Development_Chapter 13 (Final)

View Set

Chapter 6 - Modern Social Problems

View Set

Chapter 25 - Fluid, Electrolyte, Acid-Base Balance

View Set